Paeds EMQs Flashcards

1
Q

Steven has just had his first birthday party. During his party he commando-crawled with great speed, although he cannot walk. He managed to pick off all the Smarties (round chocolate sweets) from his birthday cake. He can say two words with meaning. After his birthday party, he impressed his guests by waving goodbye.

Which area of Steven’s development is delayed?

Gross motor
Fine motor and vision
Speech and hearing
Social, emotional and behavioural development
None of the above
A

None of the above

He has therefore achieved normal milestones for a 12-month-old.

How well did you know this?
1
Not at all
2
3
4
5
Perfectly
2
Q

Gerald is a 16-month-old boy who has not yet said his first word, and does not babble much. His mother believes he does not hear well because he doesn’t startle when a door slams or show any response to his name. His development is otherwise normal.

Which test would be best to assess Gerald’s hearing?

Evoked otoacoustic emission
Auditory brainstem response audiometry
Distraction hearing test
Visual reinforcement audiometry
Speech discrimination testing
A

Visual reinforcement audiometry

This is the most reliable test for a child Gerald’s age. The test requires an assistant to play with the child and keep their attention. Behind a soundproof window, another assistant will play sounds through a loudspeaker at particular frequencies. When the child turns around to the noise, a glass-fronted box with a toy inside that was previously dark lights up as visual reinforcement to reward the child for turning round.

How well did you know this?
1
Not at all
2
3
4
5
Perfectly
3
Q

Evie is 10 days old and was born in London. Her health visitor reviews her and her parents at home. She is feeding well and has a normal examination except that she has a squint. The health visitor mentions to parents she will keep this under review.

At what age does Evie need to be referred to an ophthalmologist if the squint is still present?

2 weeks
6 weeks
12 weeks
8 months
12 months
A

12 weeks

A newborn may appear to squint when looking at nearby objects because their eyes over-converge. By 6 weeks, the eyes should move together when following an object, and by 12 weeks there should be no squint present.

How well did you know this?
1
Not at all
2
3
4
5
Perfectly
4
Q

Sophie is a well 8-week-old baby who was born at term. She has come for a routine developmental check.

Which of the following would you not expect her to be able to do?

Raise her head when lying prone
Smile
Fix and follow a face or interesting object
Reach out and grasp an object
Quieten to a loud noise
A

Reach out and grasp an object

An 8-week-old infant will be able to do the remaining 4 activities, but still only has a primitive grasp reflex, so will not be able to voluntarily grasp an object. She will only be able to grasp what is placed in her hand.

How well did you know this?
1
Not at all
2
3
4
5
Perfectly
5
Q

Joanna is an active toddler. She is just being potty-trained, and has had several days where she has remained dry. She enjoys pulling her clothes off to use the potty, but cannot dress herself again. She enjoys playing by pretending to make her mother a cup of tea, but does not play well with her older siblings, as she has not yet learnt how to take turns. She is very bossy, and demands things by saying ‘give me’ or ‘me drink’. She can build a tower of six blocks, and enjoys running and climbing on furniture.

What developmental age is Joanna?

12 months
18 months
24 months
2 1/2 years
3 years
A

24 months

Joanna is dry by day, can undress and has symbolic play. She is not yet playing interactively; she will learn this at about 3 years of age. She is constructing two-word sentences. She constructs a tower of six blocks and can run.

How well did you know this?
1
Not at all
2
3
4
5
Perfectly
6
Q

Cordelia is a 4-month-old baby who is assessed by her general practitioner because of constant crying and poor feeding. She is fed by bottle on infant formula. Her mother tearfully complains that she is finding it very difficult to cope. She also has a 20-month-old boy who has recently been referred to the speech and language therapist because of language delay. She used to work as a solicitor, and her partner is a company director who often travels abroad. Charlotte’s developmental, growth and physical examination is normal.

What is the likely cause of Cordelia’s problems?

An inherited genetic condition
Gastro-oesophageal reflux
Maternal postnatal depression and stress
Pyloric stenosis
Down syndrome
A

Maternal postnatal depression and stress

This may be detrimental to Cordelia’s development, as infants are totally dependent on their main caregiver. Her older brother may have speech delay because he is not receiving enough stimulation to develop his own language skills or there may be an underlying problem causing his speech delay

How well did you know this?
1
Not at all
2
3
4
5
Perfectly
7
Q

At what age will children achieve the milestones described in the scenarios (median age)? Each age can be used once, more than once, or not at all.

Herbert can transfer objects from one hand to the other whilst sitting without support and with a straight back.

6 weeks
6 months
8 months
10 months
12 months
18 months
2 yrs
3 yrs
4 yrs
5yrs
A

8 months

How well did you know this?
1
Not at all
2
3
4
5
Perfectly
8
Q

At what age will children achieve the milestones described in the scenarios (median age)? Each age can be used once, more than once, or not at all.

Rita’s father is thrilled that she has just said her first word – says ‘dada’ to her father only.

6 weeks
6 months
8 months
10 months
12 months
18 months
2 yrs
3 yrs
4 yrs
5yrs
A

10 months

How well did you know this?
1
Not at all
2
3
4
5
Perfectly
9
Q

At what age will children achieve the milestones described in the scenarios (median age)? Each age can be used once, more than once, or not at all.

Grace enjoys drawing. She has just learnt to copy drawing a square, and can build steps using blocks after being shown.

6 weeks
6 months
8 months
10 months
12 months
18 months
2 yrs
3 yrs
4 yrs
5yrs
A

4 yrs

How well did you know this?
1
Not at all
2
3
4
5
Perfectly
10
Q

At what age will children achieve the milestones described in the scenarios (median age)? Each age can be used once, more than once, or not at all.

Anthony has a friend at nursery, and they enjoy playing with toy cars together.

6 weeks
6 months
8 months
10 months
12 months
18 months
2 yrs
3 yrs
4 yrs
5yrs
A

3 yrs

How well did you know this?
1
Not at all
2
3
4
5
Perfectly
11
Q

At what age will children achieve the milestones described in the scenarios (median age)? Each age can be used once, more than once, or not at all.

Rosette can build a three-cube tower and can point to her nose.

6 weeks
6 months
8 months
10 months
12 months
18 months
2 yrs
3 yrs
4 yrs
5yrs
A

12 months

How well did you know this?
1
Not at all
2
3
4
5
Perfectly
12
Q

At what age will children achieve the milestones described in the scenarios (median age)? Each age can be used once, more than once, or not at all.

Peace has just taken her first steps!

6 weeks
6 months
8 months
10 months
12 months
18 months
2 yrs
3 yrs
4 yrs
5yrs
A

12 months

How well did you know this?
1
Not at all
2
3
4
5
Perfectly
13
Q

At what age will children achieve the milestones described in the scenarios (median age)? Each age can be used once, more than once, or not at all.

Ivan can tie his shoe laces all by himself.

6 weeks
6 months
8 months
10 months
12 months
18 months
2 yrs
3 yrs
4 yrs
5yrs
A

5yrs

How well did you know this?
1
Not at all
2
3
4
5
Perfectly
14
Q

At what age will children achieve the milestones described in the scenarios (median age)? Each age can be used once, more than once, or not at all.

Rinah can follow her mother’s two-step commands, such as ‘fetch your red shoes from the cupboard’.

6 weeks
6 months
8 months
10 months
12 months
18 months
2 yrs
3 yrs
4 yrs
5yrs
A

3 yrs

How well did you know this?
1
Not at all
2
3
4
5
Perfectly
15
Q

At what age will children achieve the milestones described in the scenarios (median age)? Each age can be used once, more than once, or not at all.

Lizzie has just started crawling.

6 weeks
6 months
8 months
10 months
12 months
18 months
2 yrs
3 yrs
4 yrs
5yrs
A

8 months

How well did you know this?
1
Not at all
2
3
4
5
Perfectly
16
Q

At what age will children achieve the milestones described in the scenarios (median age)? Each age can be used once, more than once, or not at all.

Blessing’s mother is so pleased; her baby has just learnt to smile when she smiles at her!

6 weeks
6 months
8 months
10 months
12 months
18 months
2 yrs
3 yrs
4 yrs
5yrs
A

6 weeks

How well did you know this?
1
Not at all
2
3
4
5
Perfectly
17
Q

At which of these ages is the following action (screening, examination, health promotion activity) taken in the child health surveillance and promotion programme in the United Kingdom? Each answer can be used once, more than once, or not at all.

Hearing test using audiometry

Newborn
5-6 days
12 days
8 weeks
3 months
4 months
8 months
12 months
13 months
2-3 years
4-5 years (pre-school)
5 years (school entry)
A

13 months

How well did you know this?
1
Not at all
2
3
4
5
Perfectly
18
Q

At which of these ages is the following action (screening, examination, health promotion activity) taken in the child health surveillance and promotion programme in the United Kingdom? Each answer can be used once, more than once, or not at all.

An orthoptist assessment for visual impairment

Newborn
5-6 days
12 days
8 weeks
3 months
4 months
8 months
12 months
13 months
2-3 years
4-5 years (pre-school)
5 years (school entry)
A

4-5 years (pre-school)

How well did you know this?
1
Not at all
2
3
4
5
Perfectly
19
Q

At which of these ages is the following action (screening, examination, health promotion activity) taken in the child health surveillance and promotion programme in the United Kingdom? Each answer can be used once, more than once, or not at all.

Guthrie test (biochemical screening test)

Newborn
5-6 days
12 days
8 weeks
3 months
4 months
8 months
12 months
13 months
2-3 years
4-5 years (pre-school)
5 years (school entry)
A

5-6 days

How well did you know this?
1
Not at all
2
3
4
5
Perfectly
20
Q

At which of these ages is the following action (screening, examination, health promotion activity) taken in the child health surveillance and promotion programme in the United Kingdom? Each answer can be used once, more than once, or not at all.

First routine immunisation with 5 in 1 immunisation - DTaP/IPV, Hib, PCV (diphtheria, tetanus, pertussis, polio, Haemophilus influenza, pneumococcal)

Newborn
5-6 days
12 days
8 weeks
3 months
4 months
8 months
12 months
13 months
2-3 years
4-5 years (pre-school)
5 years (school entry)
A

8 weeks

How well did you know this?
1
Not at all
2
3
4
5
Perfectly
21
Q

At which of these ages is the following action (screening, examination, health promotion activity) taken in the child health surveillance and promotion programme in the United Kingdom? Each answer can be used once, more than once, or not at all.

Hearing test using evoked otoacoustic emission or auditory brainstem response audiometry

Newborn
5-6 days
12 days
8 weeks
3 months
4 months
8 months
12 months
13 months
2-3 years
4-5 years (pre-school)
5 years (school entry)
A

Newborn

How well did you know this?
1
Not at all
2
3
4
5
Perfectly
22
Q

At which of these ages is the following action (screening, examination, health promotion activity) taken in the child health surveillance and promotion programme in the United Kingdom? Each answer can be used once, more than once, or not at all.

Advice on reducing the risk of sudden infant death syndrome by ‘Back to sleep’, avoiding overheating and avoiding parental smoking

Newborn
5-6 days
12 days
8 weeks
3 months
4 months
8 months
12 months
13 months
2-3 years
4-5 years (pre-school)
5 years (school entry)
A

Newborn

How well did you know this?
1
Not at all
2
3
4
5
Perfectly
23
Q

At which of these ages is the following action (screening, examination, health promotion activity) taken in the child health surveillance and promotion programme in the United Kingdom? Each answer can be used once, more than once, or not at all.

First MMR (measles/mumps/rubella) immunisation

Newborn
5-6 days
12 days
8 weeks
3 months
4 months
8 months
12 months
13 months
2-3 years
4-5 years (pre-school)
5 years (school entry)
A

13 months

How well did you know this?
1
Not at all
2
3
4
5
Perfectly
24
Q

Jonathan is 4 years old and lives in a small village in southern England. He attends a paediatric outpatient clinic with his grandmother who is his legal guardian. She is concerned that he only seems to like to play with his toy train and insists on watching the same DVD every night before he goes to bed. He attends nursery where he plays with the toys but not with other children. His behaviour can be very difficult to manage at times. He does not say any words whereas the grandmother’s children were speaking in sentences at his age. On examination you notice he does not make eye contact with you and pushes his toy train back and forth on the floor. The rest of his examination is normal.

What is the most likely diagnosis?

ADHD
Autistic spectrum disorder
Developmental coordination disorder (dyspraxia)
Asperger syndrome
Expressive language disorder
A

Autistic Spectrum Disorder

Autism is a triad of impaired social interaction, speech and language disorder and ritualistic and repetitive behaviour.

How well did you know this?
1
Not at all
2
3
4
5
Perfectly
25
At what age does autistic spectrum disorder usually become evident? ``` 0-12 months 12-24 months 2-4yrs 4-8yrs >8yrs ```
2-4yrs Autistic spectrum disorder usually presents at this age because this is when language and social skills rapidly develop.
26
At what age would you expect the clinical features of cerebral palsy to become evident? ``` 0-12 months 12-24 months 2-4 years 4-8yrs >8yrs ```
0-12 months Cerebral palsy most often presents during this time when acquisition of motor skills occurs most rapidly.
27
Fortunate is an 8-month-old black African girl who was born at term. She is seen in the paediatric outpatient department. She can roll over. She does not crawl. She can say ‘dada’, but says it to everyone not just her father. She reaches out and grasps objects with her left hand but not with her right, and puts objects in her mouth. She smiles, but is not able to wave bye-bye. Select the option that concerns you most about Fortunate’s development. ``` Unable to crawl Left hand preference Unable to use sounds discriminately to parents Unable to wave bye-bye None of the above ```
Left hand preference Fortunate has developed a preference for using her left hand at 8 months. Development of a hand preference before 1 year of age is abnormal, and she needs to be assessed for hemiplegic cerebral palsy affecting her right side.
28
Gloria is a 19-month-old white British girl whose health visitor is concerned because she is still only babbling and says no distinct words. She is able to walk, scribbles with crayons and feeds herself with a spoon. What is the most appropriate first action? Hearing test Assessment by a team specialising in autistic spectrum disorders Reassure the health visitor Refer to ENT surgeon Refer to a paediatrician for a full developmental assessment
Hearing test Speech delay can be due to hearing impairment, and this should be assessed first prior to referring her to a specialist.
29
Which of these investigations would you choose to confirm the diagnosis of developmental delay in the children described in the following scenarios? Each investigation can be used once, more than once, or not at all. Clarissa, a cheerful 20-month-old white British girl, is referred to the child development clinic by her health visitor because she is not yet walking. She was born at term with no complications. She learnt to sit without support at 10 months, and is able to crawl, although she drags her right leg behind her. Her mother says that she has always been left-handed. Examination revealed reduced power and reflexes but increased tone in the right arm. ``` Chromosome karyotype DNA FISH analysis TFTs Maternal amino acids for raised phenylalanine Creatine kinase Blood lactate Congenital infection screen Cranial US CT or MRI of the brain Electroencephalography (EEG) ```
CT or MRI of the brain
30
Which of these investigations would you choose to confirm the diagnosis of developmental delay in the children described in the following scenarios? Each investigation can be used once, more than once, or not at all. Darren is a 3-year-old boy who has difficulty climbing stairs. He always needs to hold on to the railings or to have a supporting hand. He walked unsupported at 14 months. His development is otherwise normal. On examination the power in his legs is reduced, he is somewhat hypotonic but the reflexes are normal. ``` Chromosome karyotype DNA FISH analysis TFTs Maternal amino acids for raised phenylalanine Creatine kinase Blood lactate Congenital infection screen Cranial US CT or MRI of the brain Electroencephalography (EEG) ```
Creatine kinase
31
Which of these investigations would you choose to confirm the diagnosis of developmental delay in the children described in the following scenarios? Each investigation can be used once, more than once, or not at all. Dorcus is a 9-month-old Ugandan infant who attends clinic because of unusual movements. She has developed episodes of suddenly throwing her head and arms forward. These occur in repetitive bursts. She was able to sit and babble but has stopped doing so. ``` Chromosome karyotype DNA FISH analysis TFTs Maternal amino acids for raised phenylalanine Creatine kinase Blood lactate Congenital infection screen Cranial US CT or MRI of the brain Electroencephalography (EEG) ```
Electroencephalography (EEG)
32
Which of these investigations would you choose to confirm the diagnosis of developmental delay in the children described in the following scenarios? Each investigation can be used once, more than once, or not at all. Batar is a 1-week-old baby born at term who is seen in the ophthalmology clinic because of cataracts. He has a head circumference of 32cm (normal range) and weight of 2.3kg, mild jaundice, pallor and moderate hepatosplenomegaly. His mother thinks that he does not hear well. His mother had a mild flu-like illness during pregnancy. ``` Chromosome karyotype DNA FISH analysis TFTs Maternal amino acids for raised phenylalanine Creatine kinase Blood lactate Congenital infection screen Cranial US CT or MRI of the brain Electroencephalography (EEG) ```
Congenital infection screen
33
Which of these investigations would you choose to confirm the diagnosis of developmental delay in the children described in the following scenarios? Each investigation can be used once, more than once, or not at all. Geoffrey is a 10-month-old Afro-Caribbean baby referred to the child development clinic. His mother raised concerns because he is slower in his development than her four other children. He can sit, but only if he is propped up with cushions. He is not crawling or pulling to stand. On examination he is hypotonic, with some dysmorphic features, including upslanting palpebral fissures, there is a skin fold of the upper eyelid covering the inner corner of the eye and a flat occiput. He has no other medical problems except some vitiligo. He was born at term by normal vaginal delivery. He has been slow to feed. ``` Chromosome karyotype DNA FISH analysis TFTs Maternal amino acids for raised phenylalanine Creatine kinase Blood lactate Congenital infection screen Cranial US CT or MRI of the brain Electroencephalography (EEG) ```
Chromosome karyotype
34
Considering these types of cerebral palsy, choose the type of movement disorder to fit the scenarios below. Each type of cerebral palsy can be used once, more than once, or not at all. Ronaldo is a 6-year-old boy. He attends the outpatient department as his teacher has had some concerns. He has recently started school and has been noted to be unsteady on his feet. He has to walk and run with his legs quite wide apart to stop himself from falling over. His teacher also reports that he finds it difficult to grip a pen and to write because of unsteadiness. ``` Spastic hemiplegia Spastic diplegia Spastic quadriplegia Dyskinetic Hypotonic ataxic ```
Hypotonic ataxic
35
Considering these types of cerebral palsy, choose the type of movement disorder to fit the scenarios below. Each type of cerebral palsy can be used once, more than once, or not at all. Hassan is 3 years old and was born at 26 weeks gestation weighing 700g. He sat at 10 months, and has just started to walk. He can scribble and build a tower of 3 blocks. When you examine him, he is walking on tiptoes, and his legs ‘scissor’ when you lift him up. He is able to feed himself. ``` Spastic hemiplegia Spastic diplegia Spastic quadriplegia Dyskinetic Hypotonic ataxic ```
Spastic diplegia
36
Considering these types of cerebral palsy, choose the type of movement disorder to fit the scenarios below. Each type of cerebral palsy can be used once, more than once, or not at all. Moses is a 5-year-old boy who failed to attain his developmental milestones from shortly after birth. Currently, he cannot roll or talk, but he can smile. His mother complains it is difficult to dress him as both his arms and legs are stiff. On examination, his left and right upper and lower limbs are stiff and hyper-reflexic. He has a primitive grasp reflex in both hands. ``` Spastic hemiplegia Spastic diplegia Spastic quadriplegia Dyskinetic Hypotonic ataxic ```
Spastic quadriplegia
37
Considering these types of cerebral palsy, choose the type of movement disorder to fit the scenarios below. Each type of cerebral palsy can be used once, more than once, or not at all. Gerald is a 4-year-old boy who was born in Tanzania, and had severe jaundice as a neonate that could not be treated because of lack of medical services. He now has abnormal movements of all his limbs where he adopts and maintains unusual postures, and when he is startled by a loud noise the arm on one side straightens and the opposite arm bends. When he is asleep he is hypotonic. ``` Spastic hemiplegia Spastic diplegia Spastic quadriplegia Dyskinetic Hypotonic ataxic ```
Spastic quadriplegia
38
Considering these types of cerebral palsy, choose the type of movement disorder to fit the scenarios below. Each type of cerebral palsy can be used once, more than once, or not at all. Alan is a 3-year-old boy who developed a preference for using his left hand at 7 months of age. He learnt to sit at 9 months and walked at 20 months. When he runs, he holds his right arm flexed and limps with his right foot. On examination, his right upper and lower limbs are stiff, with increased reflexes. ``` Spastic hemiplegia Spastic diplegia Spastic quadriplegia Dyskinetic Hypotonic ataxic ```
Spastic hemiplegia
39
Which of the following health professionals involved in the care of a disabled child would most help the children described below? Frankie is a 6-month-old infant from the UK. She has always struggled to gain weight and never took her bottles of milk. When she was fed she would often have choking episodes which led to episodes of pneumonia. She subsequently needed to have a naso-gastric (NG) tube and gastrostomy so she could be fed. Her mother wants to start trying to feed her some solid food. ``` Dietician Social worker Psychologist Paediatrician Speech and language therapist Occupational therapist Physio Specialist health visitor ```
Speech and language therapist
40
Which of the following health professionals involved in the care of a disabled child would most help the children described below? Cathy is 15 years old and was in a road traffic accident. She spent a week in intensive care and needed an operation on her spine. She is currently not able to walk and has been shown how to use a wheelchair. Her parents are desperate to get her back home. They live in a town house where her bedroom is on the first floor, and therefore her father would need to carry her up the stairs. ``` Dietician Social worker Psychologist Paediatrician Speech and language therapist Occupational therapist Physio Specialist health visitor ```
Occupational therapist
41
Which of the following health professionals involved in the care of a disabled child would most help the children described below? Jake is a 20-month-old boy who burnt himself on a radiator whilst playing unsupervised. He attended the Accident and Emergency department where analgesia was given and dressings applied. He was seen by a paediatrician who performed a more detailed examination looking for signs of child abuse. There were no concerns and the child was discharged home to be seen later in the burns clinic. ``` Dietician Social worker Psychologist Paediatrician Speech and language therapist Occupational therapist Physio Specialist health visitor ```
Specialist health visitor
42
Which of the following health professionals involved in the care of a disabled child would most help the children described below? Sian is 2 years old. She is being followed-up for failure to thrive (growth faltering). All her investigations have come back normal but she is still not gaining adequate weight. She drinks a lot of dilute squash but her mother complains she won’t eat any of the food she gives her. ``` Dietician Social worker Psychologist Paediatrician Speech and language therapist Occupational therapist Physio Specialist health visitor ```
Dietician
43
Which of the following health professionals involved in the care of a disabled child would most help the children described below? Thomas is a 9-year-old boy who has suspected Asperger syndrome. He has problems interacting with his siblings and classmates. His academic performance at school is poor. He has a very strict daily routine and becomes very upset if this is broken. He sleeps poorly at night. ``` Dietician Social worker Psychologist Paediatrician Speech and language therapist Occupational therapist Physio Specialist health visitor ```
Psychologist
44
Which of the following health professionals involved in the care of a disabled child would most help the children described below? Bilal is a 6-year-old boy who has recently been diagnosed with Duchenne muscular dystrophy. This presented with weakness and easy fatiguability when walking. He finds it difficult to co-ordinate his movements. He is currently in mainstream school and needs to be able to move around the classroom. ``` Dietician Social worker Psychologist Paediatrician Speech and language therapist Occupational therapist Physio Specialist health visitor ```
Physio
45
Which of the following health professionals involved in the care of a disabled child would most help the children described below? Gloria is a 22-month-old girl whose health visitor is concerned because she is still only babbling and says no distinct words. She is able to walk, scribbles with crayons and feeds herself with a spoon. ``` Dietician Social worker Psychologist Paediatrician Speech and language therapist Occupational therapist Physio Specialist health visitor ```
Speech and language therapist
46
Which of the following health professionals involved in the care of a disabled child would most help the children described below? Adrianna is a 4-year-old girl who has recently moved from Poland to the UK. She has severe learning difficulties and attends a special nursery. She has epilepsy, which is difficult to control, and is on two different antiepileptic drugs. She continues to have seizures despite this medication. She does not have an underlying diagnosis and does not have co-ordinated care. ``` Dietician Social worker Psychologist Paediatrician Speech and language therapist Occupational therapist Physio Specialist health visitor ```
Paediatrician
47
Molly is 12 months old and needs to be admitted to the paediatric ward in the district hospital. There is one paediatric ward in the hospital. Her mother is concerned whether they will be geared to caring for such a young child. What is the most common age for children to be admitted to hospital?
48
Rhys is a 5-year-old boy from Wales who is referred to the paediatric team as he has developed weakness in his legs. Investigations reveal he has Duchenne muscular dystrophy. Who is the most appropriate person to inform the parents? ``` Sister on ward Senior doctor Junior doctor Nurse looking after patient General practitioner ```
Senior doctor Parents say that they would prefer a senior doctor, in the presence of a nurse, breaking bad news to them.
49
Daniel is a 15-year old boy from London who went to see his general practitioner as he has been tired and not ‘quite right’ for the last 2 months. The general practitioner took a full blood count to see if he was anaemic. The haematology laboratory phone you, a newly qualified doctor, at 6pm in the hospital saying they have received a full blood count on Daniel and his white cells are 200×109/L and that there are blast cells. You ring Daniel’s parents at home and tell them they need to come to the oncology ward at the hospital as the results of his blood tests are abnormal. They ask you what the abnormalities are and you tell them you will explain more when they come in. You ring the consultant who says he will come to the hospital to speak to the parents. When the parents arrive you ask them to wait 20 minutes until the consultant arrives and he will explain more. What is not ideal about the situation? You have told the parents over the phone the blood test was abnormal rather than in person You have asked Daniel to come out of hours rather than the following morning You did not answer the parent’s questions on the phone. You should have told them the diagnosis The family had to wait for the consultant, you should have told them the diagnosis You asked the family to come to an oncology ward rather than the paediatric assessment unit
You asked the family to come to an oncology ward rather than the paediatric assessment unit By telling them to come to an oncology ward you are telling them Daniel has cancer without actually explaining his diagnosis properly.
50
For each of the scenarios below select the most appropriate way to administer the medication required. Each option may be used once, more than once, or not at all. Luke is a 3-year-old boy who is seen in the paediatric assessment unit. He is eating and drinking adequately. He has no other past medical history and is not on any medications. He has a fever which is associated with tachypnoea and crepitations at the right base. His oxygen saturation is 96% in air. ``` Liquid Intravenous IM Intradermal Tablets Inhaled Nebulised Topical Subcutaneous ```
Liquid
51
For each of the scenarios below select the most appropriate way to administer the medication required. Each option may be used once, more than once, or not at all. A mother has just given birth to a baby girl in a hospital in England. The midwife wants to give her vitamin K. ``` Liquid Intravenous IM Intradermal Tablets Inhaled Nebulised Topical Subcutaneous ```
Intramuscular
52
For each of the scenarios below select the most appropriate way to administer the medication required. Each option may be used once, more than once, or not at all. Noel is a 7-day-old baby who is admitted to hospital with a fever. He is feeding 3–4 hourly from the breast and has no obvious source for his fever. He was born by vaginal delivery following a normal pregnancy. ``` Liquid Intravenous IM Intradermal Tablets Inhaled Nebulised Topical Subcutaneous ```
Intravenous
53
For each of the scenarios below select the most appropriate way to administer the medication required. Each option may be used once, more than once, or not at all. Mark is a 15-year-old boy who is seen in the emergency department with an exacerbation of wheeze. He is able to speak in short sentences but has an oxygen requirement and needs to be given salbutamol. ``` Liquid Intravenous IM Intradermal Tablets Inhaled Nebulised Topical Subcutaneous ```
Nebulised
54
For each of the children in pain listed below, select the most appropriate next step in their pain management. Each option may be used once, more than once or not at all. Noah is a 7-year-old boy. He attends the Accident and Emergency department in severe pain. He has been involved in a road traffic accident and has a compound fracture of his femur. He has had several episodes of vomiting. He is extremely agitated. He has no intravenous access. ``` Distraction Topical anaesthetic Regular oral NSAIDs IV morphine via a nurse-controlled pump IV patient controlled analgesic with morphine Regular oral paracetamol Reg. long acting morphine with rapid action oromorph for breakthrough pain Epidural Intranasal diamorphine Inhalation of nitrous oxide Reg oral weak opioid (codeine) ```
Intranasal diamorphine
55
For each of the children in pain listed below, select the most appropriate next step in their pain management. Each option may be used once, more than once or not at all. Zac is a 10-year-old boy who was diagnosed with a Ewing sarcoma. He has severe pain from metastatic disease, which is unresponsive to therapy. You ask the palliative care team to help with his management as the medications are insufficient. You have tried paracetamol and oral morphine without success. He has renal failure secondary to his chemotherapy. He has no allergies. ``` Distraction Topical anaesthetic Regular oral NSAIDs IV morphine via a nurse-controlled pump IV patient controlled analgesic with morphine Regular oral paracetamol Reg. long acting morphine with rapid action oromorph for breakthrough pain Epidural Intranasal diamorphine Inhalation of nitrous oxide Reg oral weak opioid (codeine) ```
Reg. long acting morphine with rapid action oromorph for breakthrough pain
56
For each of the children in pain listed below, select the most appropriate next step in their pain management. Each option may be used once, more than once or not at all. Fiona is 3 years old. She has come from her home in Northern Ireland to England to have a liver transplant. She is day one post-operative. ``` Distraction Topical anaesthetic Regular oral NSAIDs IV morphine via a nurse-controlled pump IV patient controlled analgesic with morphine Regular oral paracetamol Reg. long acting morphine with rapid action oromorph for breakthrough pain Epidural Intranasal diamorphine Inhalation of nitrous oxide Reg oral weak opioid (codeine) ```
IV morphine via a nurse-controlled pump
57
For each of the children in pain listed below, select the most appropriate next step in their pain management. Each option may be used once, more than once or not at all. Jake is 3 years old. He has had a hernia repair and has just come back to the ward. He is not allergic to any medicines and other than moderate asthma is fit and well. You are asked to write up some pain relief by the nurses. ``` Distraction Topical anaesthetic Regular oral NSAIDs IV morphine via a nurse-controlled pump IV patient controlled analgesic with morphine Regular oral paracetamol Reg. long acting morphine with rapid action oromorph for breakthrough pain Epidural Intranasal diamorphine Inhalation of nitrous oxide Reg oral weak opioid (codeine) ```
Regular oral paracetamol
58
For each of the children in pain listed below, select the most appropriate next step in their pain management. Each option may be used once, more than once or not at all. Charlie is a 13-month-old boy who is attending the community centre to receive his Measles, Mumps & Rubella (MMR) vaccination. He has no medical problems, has no allergies and is not currently on any medication. ``` Distraction Topical anaesthetic Regular oral NSAIDs IV morphine via a nurse-controlled pump IV patient controlled analgesic with morphine Regular oral paracetamol Reg. long acting morphine with rapid action oromorph for breakthrough pain Epidural Intranasal diamorphine Inhalation of nitrous oxide Reg oral weak opioid (codeine) ```
Distraction
59
For each of the children in pain listed below, select the most appropriate next step in their pain management. Each option may be used once, more than once or not at all. Achille is a 13-year-old black African boy who has sickle cell disease. He presents with pain in his left leg. He has already taken paracetamol without affect. He has no other medical problems and has no allergies. ``` Distraction Topical anaesthetic Regular oral NSAIDs IV morphine via a nurse-controlled pump IV patient controlled analgesic with morphine Regular oral paracetamol Reg. long acting morphine with rapid action oromorph for breakthrough pain Epidural Intranasal diamorphine Inhalation of nitrous oxide Reg oral weak opioid (codeine) ```
Regular oral NSAIDs
60
For each of the children in pain listed below, select the most appropriate next step in their pain management. Each option may be used once, more than once or not at all. Victoria is 6 months old. She attends the outpatient department for a blood test. ``` Distraction Topical anaesthetic Regular oral NSAIDs IV morphine via a nurse-controlled pump IV patient controlled analgesic with morphine Regular oral paracetamol Reg. long acting morphine with rapid action oromorph for breakthrough pain Epidural Intranasal diamorphine Inhalation of nitrous oxide Reg oral weak opioid (codeine) ```
Topical anaesthetic
61
Choose from the options, which best describes the reason for the doctor’s actions in managing Tolla, an 11-year-old girl who has recently arrived in the United Kingdom with her mother. They are originally from Uganda. Tolla has been admitted to the paediatric ward with a chest infection. Her mother is HIV-positive but has not told any of her family. Her husband, Tolla’s father, died of an AIDS-related illness 6 months ago. Tolla has been tested for HIV and her test result has come back positive. Her mother does not wish her to be told, as she is still very upset about her father dying. Each option may be used once, more than once or not at all. Her mother has asked you to tell her the HIV test is negative and as the doctor you do not feel this is ethically right. ``` Non maleficence Beneficence Justice Truth-telling Duty Utility Rights Autonomy ```
Truth telling
62
Choose from the options, which best describes the reason for the doctor’s actions in managing Tolla, an 11-year-old girl who has recently arrived in the United Kingdom with her mother. They are originally from Uganda. Tolla has been admitted to the paediatric ward with a chest infection. Her mother is HIV-positive but has not told any of her family. Her husband, Tolla’s father, died of an AIDS-related illness 6 months ago. Tolla has been tested for HIV and her test result has come back positive. Her mother does not wish her to be told, as she is still very upset about her father dying. Each option may be used once, more than once or not at all. As Tolla’s doctor you feel she should be told she is HIV-positive, as you will be able to offer her more support and coping strategies if she knows more about her diagnosis. ``` Non maleficence Beneficence Justice Truth-telling Duty Utility Rights Autonomy ```
Beneficence
63
Choose from the options, which best describes the reason for the doctor’s actions in managing Tolla, an 11-year-old girl who has recently arrived in the United Kingdom with her mother. They are originally from Uganda. Tolla has been admitted to the paediatric ward with a chest infection. Her mother is HIV-positive but has not told any of her family. Her husband, Tolla’s father, died of an AIDS-related illness 6 months ago. Tolla has been tested for HIV and her test result has come back positive. Her mother does not wish her to be told, as she is still very upset about her father dying. Each option may be used once, more than once or not at all. As Tolla’s doctor you feel she should not be told the diagnosis as this will potentially expose her and her family to more harm due to the stigma attached to having HIV. The knowledge that she is HIV-positive may also affect her self-esteem. ``` Non maleficence Beneficence Justice Truth-telling Duty Utility Rights Autonomy ```
Non-maleficence
64
Choose from the options, which best describes the reason for the doctor’s actions in managing Tolla, an 11-year-old girl who has recently arrived in the United Kingdom with her mother. They are originally from Uganda. Tolla has been admitted to the paediatric ward with a chest infection. Her mother is HIV-positive but has not told any of her family. Her husband, Tolla’s father, died of an AIDS-related illness 6 months ago. Tolla has been tested for HIV and her test result has come back positive. Her mother does not wish her to be told, as she is still very upset about her father dying. Each option may be used once, more than once or not at all. The hospital manager has asked you to not start antiretroviral therapy as the family do not have legal status to stay in the UK and cannot afford the medication. You start the therapy against this advice. ``` Non maleficence Beneficence Justice Truth-telling Duty Utility Rights Autonomy ```
Duty
65
Choose from the options, which best describes the reason for the doctor’s actions in managing Tolla, an 11-year-old girl who has recently arrived in the United Kingdom with her mother. They are originally from Uganda. Tolla has been admitted to the paediatric ward with a chest infection. Her mother is HIV-positive but has not told any of her family. Her husband, Tolla’s father, died of an AIDS-related illness 6 months ago. Tolla has been tested for HIV and her test result has come back positive. Her mother does not wish her to be told, as she is still very upset about her father dying. Each option may be used once, more than once or not at all. As Tolla’s doctor you feel she has a right to be involved in her own treatment and therefore should be told she is HIV-positive. ``` Non maleficence Beneficence Justice Truth-telling Duty Utility Rights Autonomy ```
Autonomy
66
For each of the following patients seen in the Accident and Emergency department select the most appropriate next step in the management plan. Each option may be used once, more than once, or not at all. Mohammed, aged 2 months, is found by his mother to be pale and floppy in his cot. The paramedics are giving bag and valve mask ventilation when he arrives in the resuscitation room. His heart rate is 40 beats/minute. ``` Airway opening manoeuvres Bag and mask ventilation High-flow oxygen IV access IV fluid Check blood glucose Check pupils Check conscious level (AVPU) Secondary survey Commence cardiac compressions using hands encircling method Commence cardiac compressions using one hand on the sternum Commence cardiac compressions using both hands ```
Commence cardiac compressions using hands encircling method
67
For each of the following patients seen in the Accident and Emergency department select the most appropriate next step in the management plan. Each option may be used once, more than once, or not at all. Kelsey, a 2-year-old girl, is found unconscious in the garden. When she is bought into the resuscitation room she is gasping and moaning. ``` Airway opening manoeuvres Bag and mask ventilation High-flow oxygen IV access IV fluid Check blood glucose Check pupils Check conscious level (AVPU) Secondary survey Commence cardiac compressions using hands encircling method Commence cardiac compressions using one hand on the sternum Commence cardiac compressions using both hands ```
Airway opening manoeuvres
68
For each of the following patients seen in the Accident and Emergency department select the most appropriate next step in the management plan. Each option may be used once, more than once, or not at all. Daniel has been playing football at his friend’s house. He has been brought to Accident and Emergency as he has become confused and is sweaty. He walks into the department. ``` Airway opening manoeuvres Bag and mask ventilation High-flow oxygen IV access IV fluid Check blood glucose Check pupils Check conscious level (AVPU) Secondary survey Commence cardiac compressions using hands encircling method Commence cardiac compressions using one hand on the sternum Commence cardiac compressions using both hands ```
Check blood glucose
69
For each of the following patients seen in the Accident and Emergency department select the most appropriate next step in the management plan. Each option may be used once, more than once, or not at all. Aisha, a 3-year-old girl, is bought to hospital by the paramedics as she has had a seizure. She is receiving high-flow oxygen, her breathing is regular, and the cardiac monitor shows a heart rate of 100 beats/minute. She is unresponsive to painful stimuli, as she does not flinch when her blood glucose is tested. ``` Airway opening manoeuvres Bag and mask ventilation High-flow oxygen IV access IV fluid Check blood glucose Check pupils Check conscious level (AVPU) Secondary survey Commence cardiac compressions using hands encircling method Commence cardiac compressions using one hand on the sternum Commence cardiac compressions using both hands ```
Check pupils
70
For each of the following patients seen in the Accident and Emergency department select the most appropriate next step in the management plan. Each option may be used once, more than once, or not at all. Nathaniel, a 4-year-old boy, is brought to hospital with shortness of breath. He is able to talk but has oxygen saturation of 90%. His capillary refill time is less than 2 seconds. ``` Airway opening manoeuvres Bag and mask ventilation High-flow oxygen IV access IV fluid Check blood glucose Check pupils Check conscious level (AVPU) Secondary survey Commence cardiac compressions using hands encircling method Commence cardiac compressions using one hand on the sternum Commence cardiac compressions using both hands ```
High-flow oxygen
71
For each of the following patients select the most likely poison. Each option may be used once, more than once or not at all. Adrian is a 3-year-old who has been staying with his grandmother over the weekend. He is bought to A&E by his grandmother who is worried he has taken one of her pills. He complains of a ‘funny’ feeling in his chest. On examination he has an irregular heart beat and an ECG reveals an arrhythmia. ``` Alcohol Salicylates Petroleum distillates Iron Disc battery ingestion Digoxin Paracetamol ```
Digoxin
72
For each of the following patients select the most likely poison. Each option may be used once, more than once or not at all. Jacob is a 15-year-old boy who is brought to hospital by ambulance. He was found in the local park by paramedics after one of his friends phoned them to say he was in trouble. When the paramedics arrived there was no one else with him. He is not coherent and unable to walk. The blood glucose level is low. ``` Alcohol Salicylates Petroleum distillates Iron Disc battery ingestion Digoxin Paracetamol ```
Alcohol
73
For each of the following patients select the most likely poison. Each option may be used once, more than once or not at all. Ruby is a 15-year-old girl who attends the Accident and Emergency department with her father as she has ‘turned yellow’. When you ask her if she has taken any medicines, she says no. She is being bullied at school, which has been going on for over a year now. On examination, she is jaundiced and her liver function tests and clotting are both deranged. ``` Alcohol Salicylates Petroleum distillates Iron Disc battery ingestion Digoxin Paracetamol ```
Paracetamol
74
For each of the following patients select the most likely poison. Each option may be used once, more than once or not at all. Callum, a 15-year-old boy, is bought into the Accident and Emergency room with his mother. She found him in his bedroom with some pills by his bed. He is disorientated and hyperventilating. ``` Alcohol Salicylates Petroleum distillates Iron Disc battery ingestion Digoxin Paracetamol ```
Salicylates
75
For each of the following patients select the most likely poison. Each option may be used once, more than once or not at all. Syam, a 2-year-old boy, comes to the Accident and Emergency department with his father. His mother noticed that his stools had become black. Syam lives at home with his parents, grandparents and two siblings. Syam’s mother is currently pregnant. On examination, Syam is cardiovascularly stable. ``` Alcohol Salicylates Petroleum distillates Iron Disc battery ingestion Digoxin Paracetamol ```
Iron
76
For each of the case summaries below, select the most likely genetic diagnosis from the list. Each answer can be used once, more than once, or not at all. Rafael is a 6-year-old boy. He is referred to the paediatric outpatient department by the school nurse. His class teacher reports that he finds most learning activities very difficult. His behaviour is difficult to manage in class. On examination you note that he has a large head which plots on the 99th centile. His height and weight are on the 25th centile. He has a long face and large ears. The remainder of examination is normal. ``` Down syndrome Edward syndrome Patau syndrome Turner syndrome Achondroplasia Neurofibromatosis Cystic fibrosis Duchenne Fragile X syndrome Prader-Willi syndrome Angelman syndrome Williams syndrome Klinefelter's syndrome ```
Fragile X syndrome X-linked recessive disorder. Unstable mutations caused by unstable expansions of trinucleotide repeat sequences inherited in Mendelian fashion. Moderate–severe learning difficulty (IQ 20–80, mean 50) Macrocephaly Macro-orchidism – postpubertal Characteristic facies – long face, large everted ears, prominent mandible and broad forehead, most evident in affected adults Other features – mitral valve prolapse, joint laxity, scoliosis, autism, hyperactivity.
77
For each of the case summaries below, select the most likely genetic diagnosis from the list. Each answer can be used once, more than once, or not at all. Amelia is a 14-year-old girl who has not started puberty. Her friends at school have all started their periods but she hasn’t even started any breast development. She is an otherwise well girl with no other medical complaints. Her mother mentions she had puffy hands and feet as a baby. On examination you note that she is short (s syndrome
Turner syndrome 45, XO. Usually (>95%), Turner syndrome results in early miscarriage and is increasingly detected by ultrasound antenatally when fetal oedema of the neck, hands or feet or a cystic hygroma may be identified. In live-born females, the incidence is about 1 in 2500. ``` Lymphoedema of hands and feet in neonate, which may persist Spoon-shaped nails Short stature – a cardinal feature Neck webbing or thick neck Wide carrying angle (cubitus valgus) Widely spaced nipples Congenital heart defects (particularly coarctation of the aorta) Delayed puberty Ovarian dysgenesis resulting in infertility, although pregnancy may be possible with in vitro fertilisation (IVF) using donated ova Hypothyroidism Renal anomalies Pigmented moles Recurrent otitis media Normal intellectual function in most. ```
78
For each of the case summaries below, select the most likely genetic diagnosis from the list. Each answer can be used once, more than once, or not at all. Roger is a 12-year-old boy. He is referred to the paediatric outpatient department because he has developed breasts. His class teacher reports that his behaviour is difficult to manage in class. On examination you note that he is very tall and slim, with his height falling on the 99th centile and his weight on the 40th centile. He has no signs of puberty and on assessing his scrotum you note he has small firm testicles. ``` Down syndrome Edward syndrome Patau syndrome Turner syndrome Achondroplasia Neurofibromatosis Cystic fibrosis Duchenne Fragile X syndrome Prader-Willi syndrome Angelman syndrome Williams syndrome Klinefelter's syndrome ```
Klinefelter's syndrome 47, XXY Infertility – most common presentation Hypogonadism with small testes Pubertal development may appear normal (some males benefit from testosterone therapy) Gynaecomastia in adolescence Tall stature Intelligence usually in the normal range, but some have educational and psychological problems.
79
For each of the case summaries below, select the most likely genetic diagnosis from the list. Each answer can be used once, more than once, or not at all. Cyril is a 2-day-old infant born to a mother who is a refugee, and had no antenatal care. He was born at term by a normal vaginal delivery. Examination at birth revealed a cleft lip and palate, a small area on his scalp where there is no skin and six toes on both feet. He is fed by a nasogastric tube as he has no suck reflex. He has a loud systolic heart murmur. ``` Down syndrome Edward syndrome Patau syndrome Turner syndrome Achondroplasia Neurofibromatosis Cystic fibrosis Duchenne Fragile X syndrome Prader-Willi syndrome Angelman syndrome Williams syndrome Klinefelter's syndrome ```
Patau syndrome ``` Structural defect of brain Scalp defects Small eyes (microphthalmia) and other eye defects Cleft lip and palate Polydactyly Cardiac and renal malformations. ```
80
For each of the case summaries below, select the most likely mode of genetic inheritance. Mercy is a 7-month-old Black-African girl who presents to the Accident and Emergency department with fever and severe pain in her digits. She has a past medical history of being chronically tired. On examination you note that she has pale conjunctiva and mild hepatomegaly. She is febrile and appears to have an upper respiratory tract infection. Her fingers are swollen. ``` Autosomal dominant Autosomal recessive Imprinting from uniparental disomy Microdeletion Multifactoral X-linked recessive X-linked dominant ```
Autosomal recessive Sickle-cell anaemia
81
For each of the case summaries below, select the most likely mode of genetic inheritance. Bruce is a 12-year-old boy who is referred by the ophthalmology team. He presented with blurred vision. Ophthalmological examination revealed a dislocated lens. He is very disappointed, as he had been doing very well in his basketball team. He is doing well academically at school. On examination you note he is very tall. He has a high arched palate, stretch marks on his skin and lax joints. His height is well above the 99th centile and he has long arms, legs and fingers. ``` Autosomal dominant Autosomal recessive Imprinting from uniparental disomy Microdeletion Multifactoral X-linked recessive X-linked dominant ```
Autosomal dominant Marfan's
82
For each of the case summaries below, select the most likely mode of genetic inheritance. Jack is a 3-year-old boy whose father has achondroplasia. Jack is very short, has bowed legs and a very prominent forehead. His head is disproportionately large compared to his body size. ``` Autosomal dominant Autosomal recessive Imprinting from uniparental disomy Microdeletion Multifactoral X-linked recessive X-linked dominant ```
Autosomal dominant Achondroplasia
83
For each of the case summaries below, select the most likely mode of genetic inheritance. Gregor, aged 6 weeks, is vomiting his feeds. His vomit is ‘like a fountain’, and immediately afterwards he is hungry again. When examined after a vomit, a mass could be felt in the right upper quadrant of his abdomen. ``` Autosomal dominant Autosomal recessive Imprinting from uniparental disomy Microdeletion Multifactoral X-linked recessive X-linked dominant ```
Multifactorial Pyloric stenosis
84
For each of the following scenarios chose the best intervention to use. A baby is born at 41 weeks by normal vaginal delivery. There were no complications in pregnancy or labour. The midwife delivers the baby who lets out a cry immediately. Pull emergency buzzer for help Place baby directly on mothers chest if she is happy for this Warm, dry and stimulate the baby Give 5 rescue breaths to the baby Start cardiac compressions and continue with ventilation breaths Intubate the baby and give surfactant Give IV adrenaline Give IM adrenaline Give IM Vit K Place infants head in the neutral position and give chin support Place in a plastic wrapping and give 5 inflation breaths
Place baby directly on mothers chest if she is happy for this
85
For each of the following scenarios chose the best intervention to use. A 28-week gestation baby is delivered by normal vaginal delivery. The junior doctor is attending the delivery, and the baby is born before the rest of the neonatal team has arrived. On assessment of the baby, he is blue and is only making a weak respiratory effort, and has a heart rate of less than 60/minute. Pull emergency buzzer for help Place baby directly on mothers chest if she is happy for this Warm, dry and stimulate the baby Give 5 rescue breaths to the baby Start cardiac compressions and continue with ventilation breaths Intubate the baby and give surfactant Give IV adrenaline Give IM adrenaline Give IM Vit K Place infants head in the neutral position and give chin support Place in a plastic wrapping and give 5 inflation breaths
Place in a plastic wrapping and give 5 inflation breaths
86
For each of the following scenarios chose the best intervention to use. You have an emergency call to a delivery suite as the baby of a mother with gestational diabetes has shoulder dystocia. You arrive before the birth. On delivery the baby is pale, makes no respiratory effort and has a heart rate of 40/min. You give 5 inflation breathes, which give good chest rise but the heart rate remains at 40/min and with no breathing. The neonatal crash team is with you. Pull emergency buzzer for help Place baby directly on mothers chest if she is happy for this Warm, dry and stimulate the baby Give 5 rescue breaths to the baby Start cardiac compressions and continue with ventilation breaths Intubate the baby and give surfactant Give IV adrenaline Give IM adrenaline Give IM Vit K Place infants head in the neutral position and give chin support Place in a plastic wrapping and give 5 inflation breaths
Start cardiac compressions and continue with ventilation breaths
87
For each of the following scenarios chose the best intervention to use. A baby is born at 38 weeks by normal vaginal delivery. The midwife clamps the cord and places the baby on the resuscitation table. She is blue and not crying. Pull emergency buzzer for help Place baby directly on mothers chest if she is happy for this Warm, dry and stimulate the baby Give 5 rescue breaths to the baby Start cardiac compressions and continue with ventilation breaths Intubate the baby and give surfactant Give IV adrenaline Give IM adrenaline Give IM Vit K Place infants head in the neutral position and give chin support Place in a plastic wrapping and give 5 inflation breaths
Warm, dry and stimulate the baby
88
For each of the following scenarios pick the most likely cause of the jaundice. Poppy is 2 weeks old and is breast-fed. She is thriving but is jaundiced. The midwife does a bilirubin which is moderately raised at 170 μmol/L, and is nearly all unconjugated. A urine dipstick is negative. ``` Physiological Congenital hypothyroidism Biliary atresia ABO incompatibility Breast-milk jaundice Sepsis Crigler-Najjar syndrome UTI Rhesus haemolytic disease G6PD deficiency Congenital infection ```
Breast-milk jaundice
89
For each of the following scenarios pick the most likely cause of the jaundice. Dimitri is 20 hours old and is noted to be markedly jaundiced, needing intensive phototherapy. His parents are Greek-Cypriots. Maternal blood is A Rhesus positive. His blood group is A Rhesus positive. He has been breast-feeding well. On examination he is markedly jaundiced, but is alert and active. ``` Physiological Congenital hypothyroidism Biliary atresia ABO incompatibility Breast-milk jaundice Sepsis Crigler-Najjar syndrome UTI Rhesus haemolytic disease G6PD deficiency Congenital infection ```
G6PD deficiency
90
For each of the following scenarios pick the most likely cause of the jaundice. Stewart is a full-term, Caucasian baby boy, born 16 hours ago. His mother is blood group O positive. The baby is breast-feeding well but the midwife has noticed he looks jaundiced. On examination the baby is clinically well. His bilirubin was 150 μmol/L at 10 hours and he was given intensive phototherapy. 6 hours later his bilirubin is 250 μmol/L. Stewart’s blood group was identified as A positive. ``` Physiological Congenital hypothyroidism Biliary atresia ABO incompatibility Breast-milk jaundice Sepsis Crigler-Najjar syndrome UTI Rhesus haemolytic disease G6PD deficiency Congenital infection ```
ABO incompatibility
91
For each of the following scenarios pick the most likely cause of the jaundice. Alfie is a 3-week-old boy whose mother is concerned about his stools being pale. He is breast-fed. On examination he is jaundiced, has mild hepatomegaly and has only just regained his birth weight. ``` Physiological Congenital hypothyroidism Biliary atresia ABO incompatibility Breast-milk jaundice Sepsis Crigler-Najjar syndrome UTI Rhesus haemolytic disease G6PD deficiency Congenital infection ```
Biliary atresia
92
For each of the following scenarios pick the most likely cause of the jaundice. Joshua is a 12-day old baby who is noted on neonatal biochemical screening (Guthrie test) to have a raised thyroid-stimulating hormone (TSH). He is recalled to the hospital. His mother reports that he is an easy baby as he sleeps a lot, and her only concern is that he is not opening his bowels very often. On examination he is moderately jaundiced. ``` Physiological Congenital hypothyroidism Biliary atresia ABO incompatibility Breast-milk jaundice Sepsis Crigler-Najjar syndrome UTI Rhesus haemolytic disease G6PD deficiency Congenital infection ```
Congenital hypothyroidism
93
Rebecca was born 48 hours ago at term. She weighed 3.2kg. You are asked to review her on the postnatal ward as she is breathing very quickly. She is not feeding. On examination she is breathing at 68 breaths/minute and has moderate chest recession. You cannot palpate her femoral pulses. Her oxygen saturations are 85% in air. ``` RDS Pneumonia Meconium aspiration Transient tachypnoea of the newborn Heart failure Anaemia Diaphragmatic hernia Tracheo-oesphageal fistula Bronchopulmonary dysplasia Persistent pulmonary hypertension Pneumothorax Coarctation of the aorta ```
Coarctation of the aorta
94
Thomas was born at 42 weeks’ gestation, with thick, meconium-stained liquor. He was pale and had a heart rate of 60 beats/minute at delivery. His vocal cords were visualised and thick meconium suctioned. To establish breathing, 5 inflation breaths were required. He then cried and his heart rate was greater than 100 beats/minute. He has severe respiratory distress and requires mechanical ventilation in 45% oxygen to maintain his oxygen saturation. ``` RDS Pneumonia Meconium aspiration Transient tachypnoea of the newborn Heart failure Anaemia Diaphragmatic hernia Tracheo-oesphageal fistula Bronchopulmonary dysplasia Persistent pulmonary hypertension Pneumothorax Coarctation of the aorta ```
Meconium aspiration
95
Zak, a full-term baby boy, is born by elective caesarean section as his mother had two previous caesarean sections. His mother was well during pregnancy and had a normal blood glucose screen. Zak becomes tachypnoeic with indrawing between his ribs at two hours of age. ``` RDS Pneumonia Meconium aspiration Transient tachypnoea of the newborn Heart failure Anaemia Diaphragmatic hernia Tracheo-oesphageal fistula Bronchopulmonary dysplasia Persistent pulmonary hypertension Pneumothorax Coarctation of the aorta ```
Transient tachypnoea of the newborn
96
For each of the following patients with nutritional problems select the most likely diagnosis. Harry is a 3-week-old Caucasian baby who has been exclusively breast-fed by his mother. He was born at home as his mother wanted ’everything to be natural’ and declined all interventions. His birthweight was 3.4 kg. He presents to the hospital with severe rectal bleeding and shock. ``` Normal Cow's milk protein intolerance CF Failure to thrive Kwashiorkor Marasmus Obesity Vit D deficiency Vit A deficiency Vit K deficiency Vit C deficiency ```
Vit K deficiency
97
For each of the following patients with nutritional problems select the most likely diagnosis. Tanya, an 11-month-old Caucasian girl, is being monitored by her health visitor. She was born weighing 2.4 kg (0.4th centile), and has remained on the 2nd centile, now weighing 7.0 kg. Her mother is on the 5th centile and her father is on the 40th centile for height. She is well, has a good appetite, and has never needed to visit her doctor. She has no abnormal signs on examination and her development is normal. ``` Normal Cow's milk protein intolerance CF Failure to thrive Kwashiorkor Marasmus Obesity Vit D deficiency Vit A deficiency Vit K deficiency Vit C deficiency ```
Normal
98
For each of the following patients with nutritional problems select the most likely diagnosis. Jonas is an 18-month-old black African boy in KwaZulu, Natal, South Africa. He was born with a weight of 3.2 kg. He was breast-fed until 9 months of age when his sibling was born. He now mainly eats the traditional maize-based porridge which is grown on the family farm. His weight is just below the 0.4th centile. He looks thin but has a distended abdomen. There is oedema around his eyes and the top of his feet. His hair has a red tinge. ``` Normal Cow's milk protein intolerance CF Failure to thrive Kwashiorkor Marasmus Obesity Vit D deficiency Vit A deficiency Vit K deficiency Vit C deficiency ```
Kwashiorkor Kwashiorkor is another manifestation of severe protein malnutrition, in which there is generalised oedema as well as severe wasting. Because of the oedema, the weight may not be as severely reduced. Also get: a ‘flaky-paint’ skin rash with hyperkeratosis (thickened skin) and desquamation a distended abdomen and enlarged liver (usually due to fatty infiltration) angular stomatitis hair which is sparse and depigmented diarrhoea, hypothermia, bradycardia and hypotension low plasma albumin, potassium, glucose and magnesium.
99
For each of the following patients with nutritional problems select the most likely diagnosis. ``` Jamie is a 5-month-old male Caucasian infant who was born with a weight of 3.5 kg (25th centile). He was initially breast-fed and was thriving. His mother developed mastitis and so he was changed to formula milk feeds. He now weighs 5.0 kg (s milk protein intolerance CF Failure to thrive Kwashiorkor Marasmus Obesity Vit D deficiency Vit A deficiency Vit K deficiency Vit C deficiency ```
Cow's milk protein intolerance
100
For each of the following patients with nutritional problems select the most likely diagnosis. Ahmed is an 18-month-old Pakistani boy who was born in the UK with a weight of 3.2 kg. He is on a mixed diet. His height is on the 10th centile, his weight is on the 0.4th centile. He is noted to have bowing of his legs. On examination, his wrists also feel wider than normal. ``` Normal Cow's milk protein intolerance CF Failure to thrive Kwashiorkor Marasmus Obesity Vit D deficiency Vit A deficiency Vit K deficiency Vit C deficiency ```
Vit D deficiency
101
For each of the following scenarios pick the most likely cause for the vomiting. Noah is 5 weeks old and has been breast-feeding well and putting on weight. However, over the last 36 hours he has been vomiting after almost every feed. The vomit goes everywhere and he then wants to feed again. All the vomits are milky. He was born at term, birth weight 3.8 kg. ``` Gastroenteritis DKA Coeliac disease Pyloric stenosis Sepsis Malrotation Intussusception UTI GOR Cyclic vomiting syndrome Raised ICP Appendicitis Meningitis Migraine Strangulated inguinal hernia ```
Pyloric stenosis
102
For each of the following scenarios pick the most likely cause for the vomiting. Bridgitta, an 8-year-old girl, presents to her family doctor with vomiting and abdominal pain. Her vomiting only started today and she has no diarrhoea or fever. She looks unwell and dehydrated on examination and has deep rapid breathing. She is repeatedly wanting apple juice to drink. She has lost weight over the last few weeks. ``` Gastroenteritis DKA Coeliac disease Pyloric stenosis Sepsis Malrotation Intussusception UTI GOR Cyclic vomiting syndrome Raised ICP Appendicitis Meningitis Migraine Strangulated inguinal hernia ```
DKA
103
For each of the following scenarios pick the most likely cause for the vomiting. Jennifer, a 14-year-old girl, presents to the Accident and Emergency department with a severe headache for the last 6 hours mainly affecting the left side of her head. She just wants to lie still in the dark and dislikes being disturbed, but her mother is concerned as she has never had such an episode before and is normally a very lively girl who is doing well at school. She has been vomiting for the last 2 hours and can’t keep anything down and is also complaining of tummy pain. On examination she is distressed by her headache and dislikes having the examination light shone on her. Her temperature is 37.2°C. She does not have neck stiffness or papilloedema. The rest of her examination is normal. ``` Gastroenteritis DKA Coeliac disease Pyloric stenosis Sepsis Malrotation Intussusception UTI GOR Cyclic vomiting syndrome Raised ICP Appendicitis Meningitis Migraine Strangulated inguinal hernia ```
Migraine
104
For each of the following scenarios pick the most likely cause for the vomiting. Amir was born by elective caesarean section for maternal pre-eclampsia. His birthweight was 3.3 kg. He is 36 hours old. He has been starting to establish breast-feeding but has been vomiting after every feed. The vomit is noted by the midwife to be green in colour. On examination his temperature is 37.2°C. His abdomen is slightly distended. The rest of his examination is normal. ``` Gastroenteritis DKA Coeliac disease Pyloric stenosis Sepsis Malrotation Intussusception UTI GOR Cyclic vomiting syndrome Raised ICP Appendicitis Meningitis Migraine Strangulated inguinal hernia ```
Malrotation
105
For each of the following scenarios pick the most likely cause for the vomiting. James, an 8-month-old boy, is bought to the emergency department by his parents. He is having episodes of abdominal pain and is just recovering from an upper respiratory tract infection. He seems well in between, but then suddenly seems to be in pain and looks pale. He has vomited several times. On questioning he has had no blood in his stool but hasn’t opened his bowels for 24 hours. ``` Gastroenteritis DKA Coeliac disease Pyloric stenosis Sepsis Malrotation Intussusception UTI GOR Cyclic vomiting syndrome Raised ICP Appendicitis Meningitis Migraine Strangulated inguinal hernia ```
Intussusception
106
Max, aged 2 years, has had a 2-day history of fever and coryza. His mother has brought him to the Accident and Emergency department as he is crying inconsolably. She thinks his tummy is hurting him. He appears reasonably well with minimal abdominal tenderness but has an indentable mass on the left side of the abdomen. ``` Mesenteric adenitis Acute appendicitis Slipped disc Gastroenteritis Viral upper respiratory tract infection Right lower lobe pneumonia Constipation Right inguinal hernia Psychosomatic UTI DKA IBD Coeliac disease Hepatitis ```
Constipation
107
Molly, aged 10 years, is brought to the paediatric assessment room as she has had vomiting and central abdominal pain for two days. She has also had some diarrhoea. She has only had apple juice and no food for the last day. Her pain is getting worse. On examination, she has a temperature of 38.2°C, pulse 110 beats/min. She has mild dehydration. There is tenderness in the lower right abdomen, but no guarding. When asked to walk, she is unable to stand up straight because of pain. ``` Mesenteric adenitis Acute appendicitis Slipped disc Gastroenteritis Viral upper respiratory tract infection Right lower lobe pneumonia Constipation Right inguinal hernia Psychosomatic UTI DKA IBD Coeliac disease Hepatitis ```
Acute appendicitis
108
Pete, aged 4 years, is brought by his mother to the Accident and Emergency department as he is crying and saying his tummy hurts. He has had a two-day history of fever, coryza and cough. He is sitting quietly on his mother’s lap, and is reluctant to play. He has a temperature of 38.2°C, and a respiratory rate of 50 beats/min. On examination, his throat is red and he has tender cervical lymph nodes. On auscultation of his chest his breath sounds at the right base are reduced compared with the left. He complains of tenderness on palpation of the right upper quadrant of the abdomen. ``` Mesenteric adenitis Acute appendicitis Slipped disc Gastroenteritis Viral upper respiratory tract infection Right lower lobe pneumonia Constipation Right inguinal hernia Psychosomatic UTI DKA IBD Coeliac disease Hepatitis ```
Right lower lobe pneumonia
109
Max, aged 9 years, has been brought by his mother to the Accident and Emergency department as he is crying and saying his tummy hurts. He has had a two-day history of fever and coryza. He has been drinking orange juice but has only eaten some jelly and yoghurt. He has not opened his bowels. His temperature is 38.2°C. His throat is red and he has tender cervical lymph nodes. He is not dehydrated. He has mild generalized tenderness of the abdomen, with no guarding. ``` Mesenteric adenitis Acute appendicitis Slipped disc Gastroenteritis Viral upper respiratory tract infection Right lower lobe pneumonia Constipation Right inguinal hernia Psychosomatic UTI DKA IBD Coeliac disease Hepatitis ```
Mesenteric adenitis
110
Francis is 12 years old and is reviewed in the paediatric outpatient department. He has been referred as he has had abdominal pain for the last 3 months. The pain is cramp-like, all over his tummy. He also has developed diarrhoea and is getting up twice in the night to open his bowels. There has not been any blood. He no longer wants to play football in the team, and is increasingly refusing to do his homework. He has lost 1 kg in weight. There are no abnormalities on examination. A blood test shows his haemoglobin 10.1 g/dl, and a raised C-reactive protein of 85 (normal
IBD
111
For each of the following patients with an infectious disease, please select the NEXT step in management. 18-month-old Mustafa presents with 3 days of vomiting and diarrhoea. The vomiting has now settled but he continues to have about 20 loose stools each day. He has sunken eyes and appears lethargic. His extremities are warm, his capillary refill time is normal but his mucous membranes are dry. His skin turgor is reduced. ``` Antipyretic/analgesia, oral Combined TB medication HAART IV acyclovir IV antibiotics IV bolus of 20ml/kg of normal saline IV immunoglobulin IV quinine Nasogastric rehydration therapy Oral antibiotic Oral rehydration solution Oral coartem Oxygen Topical antibiotic Topical emollient ```
Oral rehydration solution
112
For each of the following patients with an infectious disease, please select the NEXT step in management. Imran is 2 years old and presents with fever and a cough for 2 days. He has a fever of 38° Celsius, respiratory rate of 25 breaths/minute and his oxygen saturations are normal. He is coryzal and has an inflamed pharynx. ``` Antipyretic/analgesia, oral Combined TB medication HAART IV acyclovir IV antibiotics IV bolus of 20ml/kg of normal saline IV immunoglobulin IV quinine Nasogastric rehydration therapy Oral antibiotic Oral rehydration solution Oral coartem Oxygen Topical antibiotic Topical emollient ```
Antipyretic/analgesia, oral
113
For each of the following patients with an infectious disease, please select the NEXT step in management. Josie is a 3-year-old girl. She has had a cough and fever now for 6 days. On examination her oxygen saturations are 92% in air. Her temperature is 39°C and she has a respiratory rate of 50 breaths/minute. She has marked indrawing between her ribs and is using her accessory muscles. On percussion it is stoney dull at her left base and you hear decreased air entry on examination. ``` Antipyretic/analgesia, oral Combined TB medication HAART IV acyclovir IV antibiotics IV bolus of 20ml/kg of normal saline IV immunoglobulin IV quinine Nasogastric rehydration therapy Oral antibiotic Oral rehydration solution Oral coartem Oxygen Topical antibiotic Topical emollient ```
IV antibiotics
114
For each of the following patients with an infectious disease, please select the NEXT step in management. Mohammed is 2 years old and presents with a fever and cough for 2 days. He is drinking sips of water but is off his food. He has a temperature of 39.5° Celsius, respiratory rate of 45 breaths/minute and his oxygen saturations are normal. On examination he has mild indrawing between his ribs. His respiratory rate is 33 breaths/minute, but he is not in shock. On auscultation there are some crackles at the left base. There is no wheeze. ``` Antipyretic/analgesia, oral Combined TB medication HAART IV acyclovir IV antibiotics IV bolus of 20ml/kg of normal saline IV immunoglobulin IV quinine Nasogastric rehydration therapy Oral antibiotic Oral rehydration solution Oral coartem Oxygen Topical antibiotic Topical emollient ```
Oral antibiotic
115
For each of the following patients with an infectious disease, please select the NEXT step in management. Michael is a 5-year-old black African boy. He has been unwell now for 6 days with a fever. He has cervical lymphadenopathy, red eyes, and an injected throat. He also has a rash and is complaining that the skin is peeling from his hands and feet. ``` Antipyretic/analgesia, oral Combined TB medication HAART IV acyclovir IV antibiotics IV bolus of 20ml/kg of normal saline IV immunoglobulin IV quinine Nasogastric rehydration therapy Oral antibiotic Oral rehydration solution Oral coartem Oxygen Topical antibiotic Topical emollient ```
IV immunoglobulin
116
Jack, a 4-month-old Caucasian infant, has rapid, laboured breathing which has been getting worse over the last 2 days. His mother is concerned as she is struggling to get him to feed. He was born at 27 weeks’ gestation, birth weight 979g and was discharged home at 3 months of age. On examination he has a temperature of 37.4°C and a respiratory rate of 60 breaths/minute. He is coughing. His chest is hyperinflated with marked intercostal recession. On auscultation there are generalised fine crackles and wheezes. ``` Acute exacerbation of asthma Bronchitis Bronchiolitis Chronic asthma Bronchopulmonary dysplasia (BPD, chronic lung disease of prematurity) Cystic fibrosis Heart failure Inhaled foreign body Laryngotracheobronchiitis Obstructive sleep apnoea Pertussis Pneumonia Retropharyngeal abscess Tracheitis Tuberculosis ```
Bronchiolitis
117
Fred is a normally well 4-year-old Caucasian boy. He has had a runny nose and fever for 3 days. He has now developed a cough and difficulty breathing. On examination his temperature is 39°C. He watches you but sits quietly on his mother’s lap. He has a respiratory rate of 55 breaths/minute. His breaths are rapid but shallow, with some mild substernal recession. There is no wheeze on auscultation, but some coarse crackles at the right base. His oxygen saturations are 91% in air. ``` Acute exacerbation of asthma Bronchitis Bronchiolitis Chronic asthma Bronchopulmonary dysplasia (BPD, chronic lung disease of prematurity) Cystic fibrosis Heart failure Inhaled foreign body Laryngotracheobronchiitis Obstructive sleep apnoea Pertussis Pneumonia Retropharyngeal abscess Tracheitis Tuberculosis ```
Pneumonia
118
Connor, a 5-month-old Caucasian boy, from a travelling family visiting from Ireland, is admitted to hospital with difficulty breathing and poor feeding. He was born at term with a birthweight of 3.6 kg (50th centile). His weight is now 5.2 kg (
Cystic fibrosis
119
Hannah is an 8-month-old Caucasian baby who has had a cough for over 2 weeks. She has now developed prolonged bouts of coughing. She has started to vomit at the end of the bout of coughing. Her temperature is 38°C. Her respiratory rate is 25 breaths/minute. On auscultation of her chest there are some scattered crackles which move position after coughing. ``` Acute exacerbation of asthma Bronchitis Bronchiolitis Chronic asthma Bronchopulmonary dysplasia (BPD, chronic lung disease of prematurity) Cystic fibrosis Heart failure Inhaled foreign body Laryngotracheobronchiitis Obstructive sleep apnoea Pertussis Pneumonia Retropharyngeal abscess Tracheitis Tuberculosis ```
Pertussis
120
Jamal, a 10-month-old Asian boy, is brought at 1 a.m. to the Accident and Emergency department because he has woken up with noisy breathing. He has had coryzal symptoms for 2 days and a barking cough for 12 hours. On examination he has a fever of 37.8°C. He is alert and watches you but clings to his mother. On crying, he has a marked inspiratory stridor. ``` Acute exacerbation of asthma Bronchitis Bronchiolitis Chronic asthma Bronchopulmonary dysplasia (BPD, chronic lung disease of prematurity) Cystic fibrosis Heart failure Inhaled foreign body Laryngotracheobronchiitis Obstructive sleep apnoea Pertussis Pneumonia Retropharyngeal abscess Tracheitis Tuberculosis ```
Laryngotracheobronchiitis
121
Jake, a 7-month-old Caucasian boy, presents with a 2-day history of fever and runny nose. During the night he has developed a harsh cough in association with noisy inspiration. On examination, you note a child with moderate stridor mainly on inspiration and mild intercostal and subcostal recession. His respiratory rate is 30 breaths/minute. He has a temperature of 37.8°C. His capillary refill time is normal. His oxygen saturation is 96% in air. ``` CPAP Inhaled Salbutamol via MDI Inhaled Salbutamol via MDI and spacer Inhaled steroids via MDI Inhaled steroids via MDI and spacer IV antibiotics Nebulised adrenaline Nebulised salbutamol Oxygen Oral antipyretic/analgesic Oral antibiotics Oral steroid Intubation and ventilation Nebulised steroid (budesonide) ```
Nebulised steroid (budesonide)
122
Kane is a 6-year-old Caucasian boy with asthma. He has become acutely breathless during school sports day. He takes inhaled steroids regularly. At the paediatric assessment unit, he is having difficulty speaking, has a respiratory rate of 35 breaths/minute and marked use of accessory muscles and chest recession. His oxygen saturation is 91% in air. ``` CPAP Inhaled Salbutamol via MDI Inhaled Salbutamol via MDI and spacer Inhaled steroids via MDI Inhaled steroids via MDI and spacer IV antibiotics Nebulised adrenaline Nebulised salbutamol Oxygen Oral antipyretic/analgesic Oral antibiotics Oral steroid Intubation and ventilation Nebulised steroid (budesonide) ```
Oxygen
123
Harry, a 12-month-old Caucasian baby, has a 2-day history of fever, coryza, cough and difficulty feeding. His temperature is 37.9°C and he appears well. His respiratory rate is normal and there are no focal signs on auscultating the chest. ``` CPAP Inhaled Salbutamol via MDI Inhaled Salbutamol via MDI and spacer Inhaled steroids via MDI Inhaled steroids via MDI and spacer IV antibiotics Nebulised adrenaline Nebulised salbutamol Oxygen Oral antipyretic/analgesic Oral antibiotics Oral steroid Intubation and ventilation Nebulised steroid (budesonide) ```
Oral antipyretic/analgesic
124
Sara, an 8-month-old Asian girl, presents with a 3-day history of being unsettled. She has been coryzal and has had a mild fever. On examining the right ear canal you note a bulging red tympanic membrane. Her respiratory rate is 25/minute and she does not have any chest recession. ``` CPAP Inhaled Salbutamol via MDI Inhaled Salbutamol via MDI and spacer Inhaled steroids via MDI Inhaled steroids via MDI and spacer IV antibiotics Nebulised adrenaline Nebulised salbutamol Oxygen Oral antipyretic/analgesic Oral antibiotics Oral steroid Intubation and ventilation Nebulised steroid (budesonide) ```
Oral antipyretic/analgesic
125
Chardonnay is a 6-year-old Caucasian girl who has asthma. Her mother smokes cigarettes and there is poor compliance with her preventative steroid therapy. She presents with a 2-day history of cough, runny nose, mild fever and ‘breathlessness’. Her mother cannot remember her previous peak-flow result. On examination she has a respiratory rate of 30 breaths/minute, mild intercostal recession and oxygen saturation of 96% in air. ``` CPAP Inhaled Salbutamol via MDI Inhaled Salbutamol via MDI and spacer Inhaled steroids via MDI Inhaled steroids via MDI and spacer IV antibiotics Nebulised adrenaline Nebulised salbutamol Oxygen Oral antipyretic/analgesic Oral antibiotics Oral steroid Intubation and ventilation Nebulised steroid (budesonide) ```
Inhaled Salbutamol via MDI and spacer
126
Jane, a previously fit and well 18-month-old girl, presents with frequent respiratory tract infections and wheeze. On examination there is a fixed and widely split second heart sound, an ejection systolic murmur best heard at the upper left sternal edge. ``` Cyanotic congenital heart disease Heart failure Aortic stenosis ASD Coarctation of the aorta Dextrocardia Mitral regurgitation Mitral stenosis Normal PDA VSD ```
ASD
127
Sarah was born at term by spontaneous vaginal delivery and went home at 8 hrs of age following a normal neonatal discharge examination. At 48 hrs of age her mother found her looking pale and was unable to wake her. She was rushed to the Accident and Emergency department. Her breathing was noted to be very shallow, her skin was cool and mottled and she was unresponsive to pain. She is resuscitated, given intravenous fluids and broad spectrum antibiotics. On re-examination, the only palpable pulse is the right brachial pulse. ``` Cyanotic congenital heart disease Heart failure Aortic stenosis ASD Coarctation of the aorta Dextrocardia Mitral regurgitation Mitral stenosis Normal PDA VSD ```
Coarctation of the aorta
128
Sunit is 24 hours old and his mother notices when she is about to breast-feed that he is blue around the mouth. On examination his tongue looks blue and there is peripheral cyanosis. His respiratory rate is 65 breaths/minute. On auscultation of the chest there is no murmur. Pulses in all 4 limbs can be palpated and are equal in volume. He is watching you and moving his arms and legs vigorously whilst you examine him. ``` Cyanotic congenital heart disease Heart failure Aortic stenosis ASD Coarctation of the aorta Dextrocardia Mitral regurgitation Mitral stenosis Normal PDA VSD ```
Cyanotic congenital heart disease
129
A 5-year-old boy presents with frequent chest infections. On examination there are bibasal crackles. His heart sounds can be heard throughout the praecordium but are louder on the right. His apex beat is palpable on the right. ``` Cyanotic congenital heart disease Heart failure Aortic stenosis ASD Coarctation of the aorta Dextrocardia Mitral regurgitation Mitral stenosis Normal PDA VSD ```
Dextrocardia
130
Nada, 5 months old, has a fever and runny nose for 2 days. On examination she has a fever of 38.3°C. Her tongue is pink. Her breathing is normal in rate and pattern, other than with a runny nose. Pulse is 160 beats/minute. Her heart sounds are normal but she has a soft systolic murmur at left sternal edge. Pulses are normal. ``` Aortic stenosis ASD Coarctation of the aorta Dextrocardia Mitral regurgitation Mitral stenosis Normal PDA VSD Pulmonary stenosis Transposition of the great arteries Tetralogy of Fallot ```
Normal
131
Debbie, a 3-month-old girl, is being reviewed in the paediatric outpatient clinic. She was referred as on her 6-week check the general practitioner heard a machinery murmur throughout the praecordium. She is very well and thriving. All peripheral pulses are present and bounding. Oxygen saturations are 96% post-ductal. ``` Aortic stenosis ASD Coarctation of the aorta Dextrocardia Mitral regurgitation Mitral stenosis Normal PDA VSD Pulmonary stenosis Transposition of the great arteries Tetralogy of Fallot ```
PDA
132
Robert, a 3-year-old boy, has had a runny nose and wheeze for 3 days. On examination his pulse is 100 beats/minute. Pulses are normal. There is an ejection systolic murmur heard loudest at the upper right sternal edge which can also be heard over the carotid arteries but not at the back. ``` Aortic stenosis ASD Coarctation of the aorta Dextrocardia Mitral regurgitation Mitral stenosis Normal PDA VSD Pulmonary stenosis Transposition of the great arteries Tetralogy of Fallot ```
Aortic stenosis
133
Anoushka, a 1-year-old girl, presents to Accident and Emergency with a respiratory tract infection. She is pink and well-perfused. There is a thrill and pansystolic murmur at the lower left sternal edge. ``` Aortic stenosis ASD Coarctation of the aorta Dextrocardia Mitral regurgitation Mitral stenosis Normal PDA VSD Pulmonary stenosis Transposition of the great arteries Tetralogy of Fallot ```
VSD
134
Raj, a previously well 14-year-old Asian boy, is noted to be jaundiced. He has recently returned to the UK from India where he was visiting relatives in a rural village. He had a 10-day diarrhoea and vomiting illness whilst in India. ``` α1-Antitryspin deficiency Autoimmune hepatitis Bacterial infection Biliary atresia Cystic fibrosis Hepatitis A Hepatitis B Hepatitis C Inflammatory bowel disease Galactosaemia (metabolic disease) Primary sclerosing cholangitis Wilson disease ```
Hepatitis A
135
Jason, a 3-week-old Caucasian boy, is still jaundiced. His mother is reassured that this is likely to be ‘breast-milk’ jaundice as she is fully breast-feeding him. He presents 3 weeks later with poor feeding, vomiting and bruising on his forehead and limbs. He has pale stools. On examination the liver is palpable 4 cms below the costal margin. ``` α1-Antitryspin deficiency Autoimmune hepatitis Bacterial infection Biliary atresia Cystic fibrosis Hepatitis A Hepatitis B Hepatitis C Inflammatory bowel disease Galactosaemia (metabolic disease) Primary sclerosing cholangitis Wilson disease ```
Biliary atresia
136
A 5-week-old southern Asian male infant presents to the paediatric assessment unit with vomiting. He has not gained weight since birth. The child is jaundiced. On examination you find an infant who is jaundiced, lethargic and floppy. Cataracts are present. ``` α1-Antitryspin deficiency Autoimmune hepatitis Bacterial infection Biliary atresia Cystic fibrosis Hepatitis A Hepatitis B Hepatitis C Inflammatory bowel disease Galactosaemia (metabolic disease) Primary sclerosing cholangitis Wilson disease ```
Galactosaemia (metabolic disease)
137
Lee is a 12-year-old Asian boy who moved with his parents from China 2 years ago. He presents with episodes of vomiting which is blood-stained. On examination he is jaundiced, malnourished and has splenomegaly. ``` α1-Antitryspin deficiency Autoimmune hepatitis Bacterial infection Biliary atresia Cystic fibrosis Hepatitis A Hepatitis B Hepatitis C Inflammatory bowel disease Galactosaemia (metabolic disease) Primary sclerosing cholangitis Wilson disease ```
Hepatitis B
138
Angel, a 2-year-old boy, presents to his general practitioner with an abdominal mass noticed by his mother on dressing him. He has no other medical problems and is not on any medication. His stool pattern is regular. He is otherwise relatively well in himself. You examine his abdomen and feel a mass in his left abdomen which does not cross the midline. There is no hepatosplenomegaly. ``` Acute lymphoblastic Leukaemia Brain tumour Neuroblastoma Nephroblastoma (Wilms tumour) Hodgkin disease Bone tumour Retinoblastoma ```
Nephroblastoma (Wilms tumour)
139
Kay, a 3-year-old girl, presents to the paediatric assessment unit as her mother is worried she is pale, tired and ‘not quite right’. She has also lost 2kg of weight in the last month. On examination the child looks unwell, has pallor and a large firm, irregular abdominal mass in the centre of her abdomen. ``` Acute lymphoblastic Leukaemia Brain tumour Neuroblastoma Nephroblastoma (Wilms tumour) Hodgkin disease Bone tumour Retinoblastoma ```
Neuroblastoma
140
Douglas, a Scottish 7-year-old boy, visits his general practitioner with his mother. He has been getting headaches over the last 3 – 5 weeks which have increased in intensity. These have woken him from sleep. His mother reports that he has recently begun vomiting in the morning. His teachers have also commented his school work is getting worse. His mother thinks this may be because his sight has deteriorated as he keeps complaining of double vision. ``` Acute lymphoblastic Leukaemia Brain tumour Neuroblastoma Nephroblastoma (Wilms tumour) Hodgkin disease Bone tumour Retinoblastoma ```
Brain tumour
141
Natalia, a 4-year-old girl, presents to her general practitioner. This is her second bad episode of tonsillitis in the same month. She is tired and looks pale and has a number of bruises on her lower legs. On examination she has pallor, scattered purpuric skin lesions and hepatosplenomegaly. ``` Acute lymphoblastic Leukaemia Brain tumour Neuroblastoma Nephroblastoma (Wilms tumour) Hodgkin disease Bone tumour Retinoblastoma ```
Acute lymphoblastic Leukaemia
142
Carla is a 4-year-old girl who is seen in the paediatric assesment unit complaining of headaches. Her mother has noticed that her eye movements are not normal. The whole family have recently had a sickness bug but Carla seems to have continued vomiting. ``` FBC Clotting screen Blood film Bone marrow aspirate Urine catcholamines CT head scan Chest X-ray PET scan USS of abdomen MRI head scan Biopsy ```
MRI head scan
143
Francis, a 2-year-old boy, is taken to his general practitioner by his father who is worried as he has become ‘cross-eyed’. Otherwise he is very well in himself and has no history of vomiting. On examination he appears to be well but has an absent red reflex in his left eye. ``` FBC Clotting screen Blood film Bone marrow aspirate Urine catcholamines CT head scan Chest X-ray PET scan USS of abdomen MRI head scan Biopsy ```
MRI head scan
144
Oscar, a 3-year-old boy, attends the emergency department as his mother is worried he has lost weight and looks pale. On examination he has a large irregular mass extending across his abdomen. His blood pressure is high. ``` FBC Clotting screen Blood film Bone marrow aspirate Urine catcholamines CT head scan Chest X-ray PET scan USS of abdomen MRI head scan Biopsy ```
Urine catcholamines
145
Solomon is a 12-year-old boy. He has recently lost weight and his ‘glands are up’. His mother reports that the ‘glands’ in his neck have been enlarged for several months now. He has no other medical problems. He has not been having any episodes of fever or night sweats. On examination he has several large, irregular, hard lymph nodes in his neck. They are all greater than 2 cm in size. You order a full blood count and blood film, which are normal. ``` FBC Clotting screen Blood film Bone marrow aspirate Urine catcholamines CT head scan Chest X-ray PET scan USS of abdomen MRI head scan Biopsy ```
Biopsy
146
Niamh, a 4-year-old girl, is taken to her general practitioner as her mother has noticed she has red urine. On further questioning she has been more tired than normal and has been complaining of abdominal pain. On examination she is pale with a left-sided abdominal mass. Her urine is red and a dipstick confirms that this is blood. ``` FBC Clotting screen Blood film Bone marrow aspirate Urine catcholamines CT head scan Chest X-ray PET scan USS of abdomen MRI head scan Biopsy ```
USS of abdomen
147
Connor, a 4-year-old boy, presents to the paediatric assessment unit with his parents. They are worried, as he seems to be very tired and complains of his legs hurting. He also seems to have a fine rash which has developed on his arms. On examination he is pale, has a petechial rash on his arms and legs and has hepatosplenomegaly. ``` FBC Clotting screen Blood film Bone marrow aspirate Urine catcholamines CT head scan Chest X-ray PET scan USS of abdomen MRI head scan Biopsy ```
Bone marrow aspirate
148
Charlie, aged 5 years, is troubled by recurrent nose bleeds, the last of which took 1½ hours to stop. He has no other medical problems. His examination is normal except for pale conjunctivae. ``` Investigation reveals: • Hb 8.6 g/dl, WBC 10.2 × 109/L, platelets 350 × 109/L • PT 16 (control 12–15 sec) • APTT 46 sec (control 25–35 sec) • Fibrinogen 2.5 g/L (normal 2–4 g/L) • Factor VIII:C low ``` ``` ITP Haemophilia A Vitamin K deficiency von Willebrand Disease (vWD) Liver disease Meningococcal disease ALL Acute myeloid leukaemia Vitamin D deficiency G6PD deficiency Infectious mononucleosis Iron deficiency anaemia Sickle cell disease Normal variant (no pathology present) β-thalassaemia major β -thalassaemia trait α-thalassaemia major α-thalassaemia trait ```
von Willebrand Disease (vWD)
149
Shlomo, is a 9-day-old Jewish boy who was born in the UK. He had a religious circumcision yesterday, but the wound will not stop bleeding. On examination he is pale and has tachycardia. There is oozing of blood around the circumcision wound. A cannula is inserted and a blood cross match sent. There is now oozing around the cannula site. Investigation reveals: • Hb 8.4 g/dl, WBC 12 × 109/L, platelet count 322 × 109/L • PT 16 sec (control 12–15 sec) • APTT > 120 sec (control 25–35 sec) ``` ITP Haemophilia A Vitamin K deficiency von Willebrand Disease (vWD) Liver disease Meningococcal disease ALL Acute myeloid leukaemia Vitamin D deficiency G6PD deficiency Infectious mononucleosis Iron deficiency anaemia Sickle cell disease Normal variant (no pathology present) β-thalassaemia major β -thalassaemia trait α-thalassaemia major α-thalassaemia trait ```
Haemophilia A
150
Melissa is a 3-year-old girl. She presents to her general practitioner with a 3–4 week history of lethargy and weight loss. On examination she is pale and has widespread bruising. She has no other medical history and is not currently on any medications. The GP orders a full blood count, which reveals: Hb 6.6 g/dl, WBC 43.2 × 109/L, platelets 50 × 109/L ``` ITP Haemophilia A Vitamin K deficiency von Willebrand Disease (vWD) Liver disease Meningococcal disease ALL Acute myeloid leukaemia Vitamin D deficiency G6PD deficiency Infectious mononucleosis Iron deficiency anaemia Sickle cell disease Normal variant (no pathology present) β-thalassaemia major β -thalassaemia trait α-thalassaemia major α-thalassaemia trait ```
ALL
151
Xevera is a 7-year-Greek boy who has is seen by a paediatrician for constipation. Routine haematological testing reveals that he is anaemic with a Hb of 10 g/dl (MCH and MCV low). He is given a course of iron therapy but his anaemia does not improve. Further testing reveals HbA2 5%, HbF 3%. ``` ITP Haemophilia A Vitamin K deficiency von Willebrand Disease (vWD) Liver disease Meningococcal disease ALL Acute myeloid leukaemia Vitamin D deficiency G6PD deficiency Infectious mononucleosis Iron deficiency anaemia Sickle cell disease Normal variant (no pathology present) β-thalassaemia major β -thalassaemia trait α-thalassaemia major α-thalassaemia trait ```
β -thalassaemia trait
152
Lola is an 8-month-old girl from Cyprus. She is referred to the paediatric department because she is clinically anaemic and has failure to thrive. On examination you note that she has a large liver and spleen. Electrophoresis reveals an absence of HbA. ``` No action required Recombinant FVIII Vitamin K IV antibiotics Oral antibiotics Chemotherapy Vit D supplementation Folic acid supplementation Iron supplementation Blood transfusion Desmopressin ```
Blood transfusion
153
Ahmed is a 4-week-old child of Somali refugees who have just fled to the UK. He was circumcised yesterday, but the wound will not stop bleeding. On examination there is oozing of blood around the circumcision wound. Investigation reveals: • Hb 12.2 g/dl, WBC 11 × 109/L, platelet count 312 × 109/L • PT 36 sec (control 12–15 sec) • APTT 25 sec (control 25–35 sec) ``` No action required Recombinant FVIII Vitamin K IV antibiotics Oral antibiotics Chemotherapy Vit D supplementation Folic acid supplementation Iron supplementation Blood transfusion Desmopressin ```
Vitamin K
154
Lizzie is a 9-year old-girl who presents to the paediatric clinic. She is known to have hereditary spherocytosis. Her mother is concerned that she is very pale. 3 weeks ago Lizzie had an upper respiratory tract infection. Her mother reports that she had a fever and was very flushed with bright red cheeks. She is otherwise well and has fully recovered from her infection. Her mother informs you she has a very good diet. A full blood count reveals a Hb of 8.8 g/dl. A film reveals a normochromic normocytic anaemia with no blast cells. ``` No action required Recombinant FVIII Vitamin K IV antibiotics Oral antibiotics Chemotherapy Vit D supplementation Folic acid supplementation Iron supplementation Blood transfusion Desmopressin ```
No action required
155
George is a 3-month-old boy. He presents to the paediatric ward with a swollen leg. He had an immunisation yesterday and there is now a large swelling at the injection site. Haematological investigation reveals: • Hb 10.2 g/dl, WBC 9 × 109/L, platelet count 312 × 109/L • PT 13 sec (control 12–15 sec) • APTT 100 sec (control 25–35 sec) There is currently no bleeding and he is haemodynamically stable. His two older brothers both suffer from a bleeding disorder but both his parents and his older sister do not. ``` No action required Recombinant FVIII Vitamin K IV antibiotics Oral antibiotics Chemotherapy Vit D supplementation Folic acid supplementation Iron supplementation Blood transfusion Desmopressin ```
Recombinant FVIII
156
Angie is 6 weeks old. She was jaundiced at 24 hours of age, when her Hb was checked and was 15 g/dl. At 2 months of age she presents with an upper respiratory tract infection. The full blood count is repeated and is now Hb 10.2 g/dl. ``` No action required Recombinant FVIII Vitamin K IV antibiotics Oral antibiotics Chemotherapy Vit D supplementation Folic acid supplementation Iron supplementation Blood transfusion Desmopressin ```
No action required
157
Olivia is a 2-year-old girl who comes to the general paediatric clinic with her exasperated mother. Olivia throws tantrums whenever her mother asks her to do something that Olivia is reluctant to do. When you ask for an example, Olivia’s mother describes breakfast time that morning, when Olivia screamed and threw her toast across the room when she was asked to stop flicking her yoghurt at her baby brother. Olivia was born at term and has no medical history. ``` Explanation, advice and reassurance Referral to psychiatrist Parenting group Drug therapy CBT Individual or group dynamic psychotherapy ```
Explanation, advice and reassurance
158
Cynthia is a 15-year-old girl whose mother brings her to her general practitioner because of headaches and abdominal pain. These episodes of pain have been present for 6–9 months. There are no symptoms or signs to suggest an organic cause, so you ask some further questions. These reveal that Cynthia is bored most of the time, stays in her room and does not want to go out with her friends as she did previously. She used to be a star student, but now is doing less well in school. She often misses school because of feeling tired. She has no suicidal ideation and she has never deliberately self-harmed. She is not on any medication. ``` Explanation, advice and reassurance Referral to psychiatrist Parenting group Drug therapy CBT Individual or group dynamic psychotherapy ```
CBT
159
Steven is a 16-year-old boy who is brought to the emergency department. His mother is distressed because Steven is complaining that the television is putting thoughts into his head, and he is becoming increasingly suspicious of their next-door neighbour, who he believes is spying on him. His mood is normal, and he is fully orientated. There is no suspicion of intoxication or medical cause for his symptoms. He has no other medical problems and is not on any medications. ``` Explanation, advice and reassurance Referral to psychiatrist Parenting group Drug therapy CBT Individual or group dynamic psychotherapy ```
Referral to psychiatrist
160
Harriet, a 5-year-old girl, is known to have diabetes mellitus type 1. She was only diagnosed 2 months ago and is on a basal bolus regime of insulin. She is running around the garden at home when her mother notices she suddenly becomes aggressive towards her brother and looks pale and not her usual self. Her mother checks her blood glucose, which is 3 mmol/L. ``` Fluid resuscitation with normal saline (0.9%) Insulin infusion IV at 0.1U/kg/hr s/c insulin IV infusion of 5% dextrose IV antibiotics IV infusion of normal saline (0.9%) IV sodium bicarbonate Diet therapy alone Oral glucose drink IM glucagon Buccal glucose gel ```
Oral glucose drink
161
Jon is 12 years old. His brother has diabetes. He has started to drink a lot of fluids and pass a lot of urine. He checked his blood glucose on his brother’s glucometer. It was 19 mmol/L. When he arrives in Accident and Emergency he is well with no signs of dehydration. Diabetes mellitus type 1 is diagnosed as his blood glucose is 21 mmol/L. A venous blood sample shows a normal pH with 2 mmol/L of ketones (within the normal range) and a HbA1c blood test result is awaited. ``` Fluid resuscitation with normal saline (0.9%) Insulin infusion IV at 0.1U/kg/hr s/c insulin IV infusion of 5% dextrose IV antibiotics IV infusion of normal saline (0.9%) IV sodium bicarbonate Diet therapy alone Oral glucose drink IM glucagon Buccal glucose gel ```
s/c insulin
162
Julie, aged 7, has diabetes mellitus. She is admitted to hospital as she has vomited on 3 occasions. She has a 2-day history of being unwell with a mild fever, sore throat and decreased appetite. Her blood glucose measurement reads ‘high’. Although she was not eating, her parents maintained her usual insulin dose. On examination her temperature is 37.5°C. She is drowsy and confused. Her pulse is 150 beats/minute, blood pressure 80/45 (low for age) and capillary refill time 3 seconds. Examination of her throat shows tonsillitis. Her blood glucose is 22 mmol/L. ``` Fluid resuscitation with normal saline (0.9%) Insulin infusion IV at 0.1U/kg/hr s/c insulin IV infusion of 5% dextrose IV antibiotics IV infusion of normal saline (0.9%) IV sodium bicarbonate Diet therapy alone Oral glucose drink IM glucagon Buccal glucose gel ```
Fluid resuscitation with normal saline (0.9%)
163
Claire is 5 years old. She presents with a swollen joint. Her knee joint has been swollen for several days. There is some mild tenderness but she continues to walk, with a slight limp. Her mother reports that 3 weeks ago she had a bout of gastroenteritis associated with a fever. There were 2 episodes of bloody diarrhoea and she vomited once. Her symptoms resolved without treatment. On examination she is afebrile. Her right knee is swollen with only mild tenderness on passive movements of the joint. There is no other medical history of note and she is not on any medications. ``` Osteomyelitis Septic arthritis Cellulitis Fractured femur Osteosarcoma Reactive arthritis JIA Transient synovitis Perches disease SUFE Rheumative fever Vit D deficiency ```
Reactive arthritis
164
The parents of Suliman from Pakistan, aged 18 months, bring him to their general practitioner. They are concerned because he has suddenly stopped walking. On examination he is miserable but apyrexial. His left thigh is swollen. There is no discoloration of the skin and it is not warm to touch. On inspection it appears that he is not moving his left foot, however he withdraws it on tickling his foot which leads him to cry. He is unable to weight-bear. He has no other medical problem and is not on any medications. ``` Osteomyelitis Septic arthritis Cellulitis Fractured femur Osteosarcoma Reactive arthritis JIA Transient synovitis Perches disease SUFE Rheumative fever Vit D deficiency ```
Fractured femur
165
Emma, aged 3½ years, presents to the Accident and Emergency department. She has a 24-hour history of a limp and pain in her left thigh. There is no history of trauma. Her temperature is 38.9 °C and her pulse is 150 beats/minute. On examination the left thigh is swollen and held flexed and abducted. She holds it as still as possible and cries if it is moved. The upper leg is slightly red and is warmer to touch than the right leg. She has other medical history and her mother has given some paracetamol, which had no effect on the pain. ``` Osteomyelitis Septic arthritis Cellulitis Fractured femur Osteosarcoma Reactive arthritis JIA Transient synovitis Perches disease SUFE Rheumative fever Vit D deficiency ```
Septic arthritis
166
Craig is 5 years old and is complaining that his back hurts. This started about 4 weeks ago and his mother tried to ignore it as there was nothing to see and ibuprofen seemed to help. However he has now started waking up at night crying out that his back is sore. He has not had a fever. His father suffers from back pain and is off work with it at the moment. On examination he is reluctant to move his back for you but there is no obvious tenderness over his spine and there is mild scoliosis. His neurological examination is normal. ``` Growing pains Osteomyelitis Osteoid osteoma - tumour ALL Hypermobility Complex regional pain syndrome Osgood-schlatter disease Chondromalacia patellae Osteochondritis dissecans Rickets Spondyolysis Scheuermann disease ```
Osteoid osteoma - tumour
167
Joseph is a 14-year-old boy who presents to his general practitioner. He complains of pain and swelling in his right knee for several weeks. This is worse after playing football. On examination you note swelling over the tibial tuberosity. He has no other medical problems and has been taking paracetamol when the pain is present. ``` Growing pains Osteomyelitis Osteoid osteoma - tumour ALL Hypermobility Complex regional pain syndrome Osgood-schlatter disease Chondromalacia patellae Osteochondritis dissecans Rickets Spondyolysis Scheuermann disease ```
Osgood-schlatter disease
168
Milly, a 6-year-old girl, is taken to her general practitioner by her mother as she has started waking up at night and complaining her legs hurt. This started about 2 months ago. She goes off to sleep okay, but wakes up at around midnight and cries out. Her mother feels she is in pain, and has to massage her legs before she will go back to sleep. On examination she looks well. Her legs look normal and she has a full range of movement around all the joints. The rest of her examination is normal. ``` Growing pains Osteomyelitis Osteoid osteoma - tumour ALL Hypermobility Complex regional pain syndrome Osgood-schlatter disease Chondromalacia patellae Osteochondritis dissecans Rickets Spondyolysis Scheuermann disease ```
Growing pains
169
Flora is a 12-year-old girl. She presents to the Accident and Emergency department for the 3rd time in 10 days. Her right ankle is extremely painful and she is unable to weight-bear, and is brought into the department in a wheelchair. She was playing netball 10 days ago when the ball landed on her foot. Ever since then it has been painful and has been getting worse. She had an X-ray which showed no fracture. On examination she has extreme tenderness over the medial aspect of her foot, even when touched very lightly with a bit of cotton wool. The right foot feels slightly colder than the left foot, but her foot pulses are normal. She refuses to move the foot and will not let you try and move it for her as it is so painful. The rest of her examination is entirely normal and she is systemically well with no fever. ``` Growing pains Osteomyelitis Osteoid osteoma - tumour ALL Hypermobility Complex regional pain syndrome Osgood-schlatter disease Chondromalacia patellae Osteochondritis dissecans Rickets Spondyolysis Scheuermann disease ```
Complex regional pain syndrome
170
Sujinder is a 14-year-old girl who is seen in the outpatient department, complaining of pains in her knees when walking upstairs. She is otherwise well. She is free of pain whilst sitting in her chair, but complains of pain in both knees when asked to stand up. On examination you notice that she has loss of the medial arch of her feet. She has no other medical problems and is not on any medication. ``` Growing pains Osteomyelitis Osteoid osteoma - tumour ALL Hypermobility Complex regional pain syndrome Osgood-schlatter disease Chondromalacia patellae Osteochondritis dissecans Rickets Spondyolysis Scheuermann disease ```
Chondromalacia patellae
171
Jack is a 14-month-old boy. He has only recently started walking. His mother brings him to his general practitioner as she is concerned that he walks with a limp. You examine his gait and he has a painless limp of his right leg. He has asymmetry of the skin folds around his right thigh and his right hip cannot be fully abducted. He has no other medical history. He was born by elective caesarean section because of breech presentation. ``` Bow legs - normal variant Bow legs - rickets DDH Perthesdisease SUFE Pectus excavatum Pectus carinatum Scoliosis Kyphosis Normal variant of childhood ```
DDH
172
Rachel, a 2-year-old girl, is seen for her developmental check by the health visitor. She is walking on her own but the health visitor notes that she has flat feet while she is walking. She tries to get Rachel to stand on her tiptoes but she cannot. When she passively extends her big toe an arch is demonstrated. ``` Bow legs - normal variant Bow legs - rickets DDH Perthesdisease SUFE Pectus excavatum Pectus carinatum Scoliosis Kyphosis Normal variant of childhood ```
Normal variant of childhood
173
Zahra is a 4-year-old Asian girl. Her mother is concerned about her leg shape. On examination you notice that on standing, the knees are very widely spaced with the feet held together. On completing the musculoskeletal examination you notice that she has swollen wrist joints. There is no other medical history of note and she is not on any medications. ``` Bow legs - normal variant Bow legs - rickets DDH Perthesdisease SUFE Pectus excavatum Pectus carinatum Scoliosis Kyphosis Normal variant of childhood ```
Bow legs - normal variant
174
Alliah is a 2½-year-old girl. She is seen in the paediatric outpatient department because her mother is worried about her walking, as she usually walks on tiptoes. She was born by normal vaginal delivery at 37 weeks. She learnt to walk at 15 months. She never crawled but did bottom shuffle. On examination of her gait she mostly walks and runs on tiptoes, and her mother confirms she has done this ever since she started walking. However, she can walk on her heels if asked to do so. General examination of her legs demonstrates normal tone and reflexes. ``` Bow legs - normal variant Bow legs - rickets DDH Perthesdisease SUFE Pectus excavatum Pectus carinatum Scoliosis Kyphosis Normal variant of childhood ```
Normal variant of childhood
175
Keith, a 15-year-old boy, attends paediatric outpatient department with his mother. Over the last 6 weeks he has become increasingly aggressive, and last week was suspended from school for throwing his book at a teacher. His mother says that this behaviour is completely out of character; he was previously a kind and hard-working boy. His school performance has also deteriorated. He also complains of an occipital headache that is worse in the morning or when he bends down. His mother sometimes finds vomit in the sink. Examination is normal except for a squint. On checking his eye movements, you find that he is unable to deviate his left eye laterally. He has no other medical problems. He has tried taking paracetamol for his headache but this has not been therapeutic. ``` Acne vulgaris Asthma Depression Epilepsy Chronic fatigue syndrome Anorexia nervosa Somatic symptoms Risk taking behaviour Pregnancy Malignancy Psychosis ```
Malignancy
176
Anika, a 15-year-old girl, presents with her mother to her general practitioner. She complains of vomiting. This is present most commonly in the morning and is associated with abdominal pain, fatigue and breast tenderness. You ask her if she has had unprotected sex, and she denies this. She has a suprapubic mass. She has no other medical problems and is not on any medications. ``` Acne vulgaris Asthma Depression Epilepsy Chronic fatigue syndrome Anorexia nervosa Somatic symptoms Risk taking behaviour Pregnancy Malignancy Psychosis ```
Pregnancy
177
Bennu, an Egyptian 14-year-old girl, is sent to the school nurse by her maths teacher because she is complaining of a headache. She complains of a headache at least once a week, and often feels tired. There is no pattern to her headaches. She has no other symptoms and is doing well at school. She has lots of friends and is a sociable teenager. There are no abnormalities on examination. She has no other medical problems and is on no medications. ``` Acne vulgaris Asthma Depression Epilepsy Chronic fatigue syndrome Anorexia nervosa Somatic symptoms Risk taking behaviour Pregnancy Malignancy Psychosis ```
Somatic symptoms
178
Gareth is a 15-year-old boy who has become increasingly withdrawn. He frequently argues with his mother, particularly when she believes he should stay at home to do his homework rather than leaving the house to meet his new group of friends. His mother believes that this new group of friends are using drugs. She is very upset that he has even been cautioned by the police, for urinating in public whilst intoxicated. He has no other medical problems. ``` Acne vulgaris Asthma Depression Epilepsy Chronic fatigue syndrome Anorexia nervosa Somatic symptoms Risk taking behaviour Pregnancy Malignancy Psychosis ```
Risk taking behaviour
179
Sam, 16 years old, is brought to his general practitioner because his parents are worried about him. They complain that he stays locked in his room and is not doing well at school. He often refuses to get out of bed because he has a headache. You speak to Sam alone. He complains of being bored all the time, and says that there is nothing he enjoys. He feels hopeless. He no longer goes out with his friends. He complains of a headache, which is of gradual onset and like a tight band around his head. He is a fan of ‘ER’, a medical drama, where one of the characters suffered from a brain tumour. He is convinced that he also has a brain tumour. He has no other medical problems. He takes paracetamol for his headache, with good effect. ``` Acne vulgaris Asthma Depression Epilepsy Chronic fatigue syndrome Anorexia nervosa Somatic symptoms Risk taking behaviour Pregnancy Malignancy Psychosis ```
Depression
180
Jane is an 11-year-old Scottish girl who presents to the outpatient department. She complains that she finds it difficult to play sports such as basketball. In particular she finds it difficult to run and coordinate her arm movements. She was born at 32 weeks’ gestation and discharged from the neonatal service at 2 years of age as her development was normal. Her neurological examination appears to be normal except when you are examining her gait. On heel-walking, her left arm moves into a flexed position. ``` Babinkski sign Brudzinski sign Chorea Clonus Dyskinesia Fogg's test Gowers sign Kernig sign Romberg test Trendelenberg sign ```
Fogg's test
181
Paolo is a 7-year-old Ecuadorian boy who presents to the Accident and Emergency department with fever. His mother reports that this has been present for the last 8 hours and that he is sleepy and wants to sleep in a dark room. He has no other past medical history. On examination you note that he is photophobic. Whilst he is lying on the couch you flex his right knee and hip to 90 degrees. On fully extending the knee, he complains of pain and arches his back. ``` Babinkski sign Brudzinski sign Chorea Clonus Dyskinesia Fogg's test Gowers sign Kernig sign Romberg test Trendelenberg sign ```
Kernig sign
182
Ahmed is a 5-year-old Bangladeshi boy who presents to the outpatient clinic with weakness. His parents report that he finds it difficult to climb the stairs at home. This has been becoming progressively worse over the preceding 6–12 months. He is playing on the carpet with his brother when his mother calls him over. He has to roll from his back onto his front in order to stand up. ``` Babinkski sign Brudzinski sign Chorea Clonus Dyskinesia Fogg's test Gowers sign Kernig sign Romberg test Trendelenberg sign ```
Gowers sign
183
Ahmed is a 5-year-old Bangladeshi boy who presents to the outpatient clinic with weakness. His parents report that he finds it difficult to climb the stairs at home. This has been becoming progressively worse over the preceding 6–12 months. He is playing on the carpet with his brother when his mother calls him over. He has to roll from his back onto his front in order to stand up. ``` Babinkski sign Brudzinski sign Chorea Clonus Dyskinesia Fogg's test Gowers sign Kernig sign Romberg test Trendelenberg sign ```
Romberg test
184
Ahmed is a 5-year-old Bangladeshi boy who presents to the outpatient clinic with weakness. His parents report that he finds it difficult to climb the stairs at home. This has been becoming progressively worse over the preceding 6–12 months. He is playing on the carpet with his brother when his mother calls him over. He has to roll from his back onto his front in order to stand up. ``` Babinkski sign Brudzinski sign Chorea Clonus Dyskinesia Fogg's test Gowers sign Kernig sign Romberg test Trendelenberg sign ```
Babinkski sign
185
Jude is a 14-year-old Israeli boy who is on treatment for generalised tonic-clonic and absence seizures. He attends a routine clinic appointment complaining of regularly spilling his tea in the morning because his arm jerks involuntarily. These jerks are worse if he has stayed up late the night before. He is doing relatively well at school. ``` Lennox-Gastaut syndrome Childhood absence epilepsy Juvenile myoclonic epilepsy Benign Rolandic epilepsy (benign epilepsy with centrotemporal spikes) Benign childhood occipital epilepsy West syndrome (infantile spasms) Benign neonatal convulsions Landau-Kleffner syndrome ```
Juvenile myoclonic epilepsy
186
Damasco is a 6-year-old Afro-Caribbean boy who presents in paediatric clinic. He has been referred because his brother, with whom he shares a room, has woken his parents early in the morning on several occasions, complaining that Damasco is making unusual sounds and drooling from his mouth. By the time his parents come to the room, Damasco is jerking his upper and lower limbs rhythmically. This lasts for 1–2 minutes. Afterwards Damasco complains of a funny sensation on the left side of his mouth, and then goes back to sleep. He is doing well at school and clinical examination is normal. He has no other medical problems. ``` Lennox-Gastaut syndrome Childhood absence epilepsy Juvenile myoclonic epilepsy Benign Rolandic epilepsy (benign epilepsy with centrotemporal spikes) Benign childhood occipital epilepsy West syndrome (infantile spasms) Benign neonatal convulsions Landau-Kleffner syndrome ```
Benign Rolandic epilepsy (benign epilepsy with centrotemporal spikes) the most common epilepsy syndrome in childhood. The cardinal features of rolandic epilepsy are infrequent, often single, focal seizures consisting of: a. unilateral facial sensorimotor symptoms (30% of patients) b. oropharyngolaryngeal manifestations (53% of patients) c. speech arrest (40% of patients), and d. hypersalivation (30% of patients)
187
Andrew is a 5-year-old boy from Scotland with severe learning difficulties who attends a special school. He is seen in his review clinic. His epilepsy is difficult to control, and he is on 2 different anti-epileptic drugs. He has several different types of seizures: sudden stiffening of his limbs causing him to fall, episodes of staring blankly ahead for up to 30 seconds before resuming previous activities, and episodes of his head dropping with a brief loss of consciousness. He is fed via a gastrostomy feed and has chronic drooling. ``` Lennox-Gastaut syndrome Childhood absence epilepsy Juvenile myoclonic epilepsy Benign Rolandic epilepsy (benign epilepsy with centrotemporal spikes) Benign childhood occipital epilepsy West syndrome (infantile spasms) Benign neonatal convulsions Landau-Kleffner syndrome ```
Lennox-Gastaut syndrome a difficult-to-treat form of childhood-onset epilepsy that most often appears between the second and sixth year of life, and is characterized by frequent seizures and different seizure types; it is often accompanied by developmental delay and psychological and behavioral problems.
188
Bosco is an active 7-year-old Afro-Caribbean boy, who was referred to the outpatient department with a history of collapse. He collapses during football practice or when he is playing outside with his siblings. This has happened at least 6 times in the last 3 months. His mother says that he suddenly becomes pale, loses consciousness, and then slowly comes round. He has no past medial history of note. His father died suddenly at the age of 34 years. He has never had a blood glucose recorded. ``` Childhood absence epilepsy Breath-holding attack Reflex anoxic seizure Syncope Migraine Benign paroxysmal vertigo Cardiac arrhythmia Non-epileptic attack disorder (pseudo seizure) Tics Intracranial haemorrhage Night terrors Tonic-clonic seizures Narcolepsy Hypoglycaemia ```
Cardiac arrhythmia
189
Emily, a normally fit and healthy 3-year-old white British girl is rushed to Accident and Emergency by her distressed mother. The child had been playing at nursery, and had banged her head against a door, and almost immediately went very pale and stiff, and had jerking movements of her arms and legs for 20 seconds. Afterwards she was groggy for a few minutes, but is now back to her usual self. This has not happened before. Her mother is very distressed and now reports feeling faint herself. The triage nurse has performed a blood glucose which is 4.1 mmol/L. ``` Childhood absence epilepsy Breath-holding attack Reflex anoxic seizure Syncope Migraine Benign paroxysmal vertigo Cardiac arrhythmia Non-epileptic attack disorder (pseudo seizure) Tics Intracranial haemorrhage Night terrors Tonic-clonic seizures Narcolepsy Hypoglycaemia ```
Reflex anoxic seizure
190
Dora is an 8-year-old Welsh girl who is attends the outpatients department. Her mother is worried because she used to be top of her class, but is not doing as well at school this year. Whilst in clinic, you notice that Dora has an episode where she suddenly stops what she is doing, stares ahead whilst flickering her eyelids for a couple of seconds, and then resumes her previous activity as if nothing had happened. You ask her to hyperventilate, and the same thing happens again. She is growing normally and is otherwise fit and well. ``` Childhood absence epilepsy Breath-holding attack Reflex anoxic seizure Syncope Migraine Benign paroxysmal vertigo Cardiac arrhythmia Non-epileptic attack disorder (pseudo seizure) Tics Intracranial haemorrhage Night terrors Tonic-clonic seizures Narcolepsy Hypoglycaemia ```
Childhood absence epilepsy
191
Rene is a lively 2-year-old boy from France who is seen in the acute paediatric assessment unit, having had an episode of turning blue and collapsing. This is not the first time that this has happened. He recovers quickly after these events. During the consultation his mother repeatedly gives him sweets to keep him calm. These episodes only occur when he is crying and this has led to his mother trying to avoid anything which will upset him. He has no other medical problems. His blood glucose today is 4.9 mmol/L. ``` Childhood absence epilepsy Breath-holding attack Reflex anoxic seizure Syncope Migraine Benign paroxysmal vertigo Cardiac arrhythmia Non-epileptic attack disorder (pseudo seizure) Tics Intracranial haemorrhage Night terrors Tonic-clonic seizures Narcolepsy Hypoglycaemia ```
Breath-holding attack
192
Jennifer, an 11-year-old white Caucasian girl, is brought to Accident and Emergency after collapsing at school. Her teacher described her standing in assembly, becoming pale and collapsing to the floor. She had a couple of jerking movements of her limbs lasting a few seconds. She returned to normal promptly. Jennifer says that she had not eaten breakfast that morning, and experienced a sensation of feeling hot, a black curtain coming in front of her eyes, sounds becoming distant, and feeling dizzy. The school nurse took a blood glucose which was 3.5 mmol/L. She has had 2 episodes similar to this in the past, but has not presented to hospital before. She has no other medical problems. ``` Childhood absence epilepsy Breath-holding attack Reflex anoxic seizure Syncope Migraine Benign paroxysmal vertigo Cardiac arrhythmia Non-epileptic attack disorder (pseudo seizure) Tics Intracranial haemorrhage Night terrors Tonic-clonic seizures Narcolepsy Hypoglycaemia ```
Syncope